Sie sind auf Seite 1von 76

LICENSURE EXAMINATION FOR TEACHERS

GENERAL EDUCATION- SCIENCE


1.

What indoor relative humidity range would probably be comfortable when the
outside temperature and humidity levels are extremely low?
A. 90 to 100%
B. 60 to 70%
C. 20 to 30%
D. 30 to 40%

2. Which of the following facts support the Big Bang Theorys explanation of the
creation of the universe?
A. The universe does not expand nor contract.
B. The universe seldom expands.
C. The universe will have background radiation.
D. The universe has no beginning or end. *
3. Which of the following is the BEST example of self-preservation?
A. A mouse runs when it sees a cat.
B. A dog barks when it sees its owner.
C. A young man decides to quit smoking.
D. A salmon swims back to the place of its birth to lay eggs.
4. Which of the
I.
II.
III.

following internal forces interrupt the external forces of erosion?


Forces that cause volcanoes
Forces that cause ocean trenches
Forces that create mountains.
A. I, II, and III *
B. I and III
C. II and III
D. I and II

5. Which location should have most nearly twelve hours of daylight and twelve
hours of darkness during December? A town that is located
A. halfway between the equator and the South Pole
B. close to the equator
C. close to the North Pole
D. close to the South Pole
6. At which time during the year does the ozone level present a particular health
threat in urban areas for people with respiratory problems?
A. Spring
B. Fall
C. Summer *
D. Winter
7. Which of the following procedures used by a farmer is NOT related directly to
preventing erosion?
A. Contour plowing around a hill
B. Planting more seeds than are necessary to yield a bountiful
crop*
C. Planting grass in gullies to act as a filter
D. Planting crops in alternate rows (strip farming)
8. A nation which loess would likely be found today is
A.
B.
C.
D.

Iceland *
United States
Japan
Ecuador

9. Which one explains why oxygen, a gas, is the largest component of the Earths
crust?

A.
B.
C.
D.

Oxygen gives Earths crust its lightness


Oxygen is the most abundant element in the world
Oxygen is capable of combining with most of the elements in
the Earths crust. *
Oxygen is needed to sustain all life on Earth.

10. Which is the BEST evidence that helium gas is lighter than air?
A. Helium has the lowest boiling point of all elements.
B. Helium atoms do not combine with other air atoms.
C. Helium-filled balloons rise in air. *
D. By volume, helium makes up only 0.0005% of air.
11. In which kingdom should MOLD be classified?
A. Protista
B. Plantae
C. Fungi*
D. Animalia
12. Which area of the brain controls feelings on the left side of a persons face?
A. The left occipital lobe
B. The left parietal lobe
C. The right parietal lobe
D. The right occipital lobe
13. Scientist also found that other stalky vegetables such as carrots also help lower
blood pressure. This statement is BEST classified as
A. experiment
B. nonessential fact
C. finding *
D. prediction

14. The Santiago family has had four children, all girls. The fifth child born is a boy.
This change is the result of
A. A conception classes taken by the parents.
B. The timing of the fertility cycles.
C. The fathers contribution of a Y chromosome. *
D. The law of averages finally catching up.
15. Which of the following should you expect to be true about cellular respiration for a
group of students who are the same age, height, and weight?
A. Athletes would tend to have higher rates of cellular
respiration than non-athletes.
B. Africans would have a higher rate of cellular respiration than
Asians.
C. Boys would have a higher rate of cellular respiration than
girls. *
D. Non-athletes would have higher rates of cellular respiration
than girls.
16. What is the function of DIFFUSION in the human body?
A. Regulates blood flow
B. Plays an insignificant role in the bodys functioning
C. Allows an even distribution of substances throughout all cells
of the body *
D. Comes into play in times of extreme illness
17. What is the MOST likely reason for a desert plant to have a few or no leaves?
A. To increase photosynthesis
B. To decrease photosynthesis
C. To increase transpiration
D. To decrease transpiration *
18. It was also discovered that the chemical 3m butylphthalide can lower the blood
pressure of rats. This statement is classified as ____________________.
A. Prediction

B.
C.
D.

Experiment
Finding *
Nonessential fact

19. Which kingdom should streptococcus be classified?


A. Protista
B. Pantae
C. Fungi
D. Monera
20. Which of the following methods can all diabetics control their condition and avoid
heart disease and blindness?
I. Regulating their intake of glucose
II. Increasing the levels of insulin on the body by taking insulin injections
III. Maintaining a regular exercise regimen to keep weight down
A. I
B. II
C. I and II
D. I and III
21. Without the process of meiosis, we can infer that offspring from sexual
reproduction would
A. have a high degree of genetic variety
B. have twice the assigned number of chromosomes
C. be identical *
D. have a number of mutations
22. Which method of reproduction provides for the most variety of offspring?
A. Cloning
B. Sexual reproduction *
C. Asexual reproduction
D. Cellular reproduction
23. What does a stick of unlit dynamite demonstrate?
A. Chemical energy *
B. Nuclear energy
C. Kinetic energy
D. Potential energy
24. The principle under
A.
B.
C.
D.

which a thermostat operates is the same when


A gas expands to fill the container in which it is held.
A pendulum swings when it is set into motion.
A chemical reaction occurs when two substances combine.
The level of mercury rises or falls in a glass tube.

25. What do you predict will happen when you bring two bar magnets closer
together?
A. They will repel each other.
B. They will create an alternating current.
C. Nothing will happen.
D. They will attract each other.
26. Which of the following BEST demonstrates the greenhouse principle?
A. A heated aquarium
B. A car with rolled-up windows *
C. A microwave oven
D. A solar battery-powered calculator

27. Which law of force and motion explains this occurrence, when a rocket is
propelled upward by the powerful downward discharge of exhaust gases?
A. Universal Gravitation
B. Action and Reaction *
C. Applied Force

D.

Inertia

28. Why does a pendulum in a grandfathers clock once set in motion continue to
swing thereby regulating the clocks movement? This is due to the law of
A. Universal Gravitation
B. Action and Reaction
C. Applied Force
D. Inertia *
29. Why does a bullet when discharged into the air eventually fall to the ground? This
is due to the Law of _______________.
A. Universal Gravitation *
B. Applied Force
C. Inertia
D. Action and Reaction
30. Which of the following will occur if a cold bottle of soda is left open on a kitchen
counter?
A. The pressure that the soda exerts on the bottle will increase.
B. The temperature of the soda will decrease.
C. The amount of dissolved carbon gas will decrease.*
D. The amount of dissolved carbon gas will remain the same.
31. In an experiment, a drop of blue ink is placed on the surface of a glass of water.
In a few minutes, the drop on ink is dispersed throughout the water, turning it
light blue. The result of the experiment proves that
A. Molecules of ink and molecules of water are in constant
motion.*
B. Heat causes the ink to disperse.
C. A new compound is formed by the combination of ink and
water.
D. Ink molecules have less density than water molecule.
32. Each of the following objects is designed to employ the buoyancy principle
EXCEPT a
A. Life preserver
B. Kite*
C. Submarine
D. canoe
33. An elderly woman suffered a strokea restriction of blood flow to the brain. If the
stroke caused the right side of her body to become temporarily paralyzed, she
most likely experienced a decreased blood flow to
A. the left side of her body
B. the front of her brain
C. the left side of her brain*
D. the right side of her brain

34. In an experiment, a vacuum is created when all air is removed from a tube. A
coin and bits of confetti are released in the vacuum at the same time. They fall at
the same rate and reach the bottom at the same time. The experiment
proves that
I. in a vacuum, the rate of acceleration is the same for all objects regardless of
weight
II. outside a vacuum, air resistance is what make objects fall at different rates
III. gravity has no effect at all on objects that fall in a vacuum.
A. I and II
B. I and III
C. I, II, and III
D. II and III
35. New peonies, perennial plants that produce snowy flowers can be propagated
from the parent plant by dividing corns that grow underground. This reproductive
form resembles a _________________.
A. bulb

B.
C.
D.

runner
seed
bud

36. Which antidote would have a similar effect if vinegar or citrus juice were
available?
A. Milk
B. Raw egg white
C. Vegetable oil
D. Water
37. Which is NOT among the hazardous effects of water pollution to health?
A. The epidemic threat of hepatitis and dysentery
B. The increased incidence of liver cancer
C. The dumping of mercury in the sea causing blindness, brain
damage, or death
D. The presence of certain bacteria in the digestive tract causing
methemoglobinemia
38. A tightly coiled spring demonstrates ____________.
A. Steam energy
B. Kinetic energy
C. Potential energy*
D. Chemical energy
39. As a representative of the Urban Poor Commission of the Association of
Regulations Superiors (ARS), which action will you most likely take to resolve the
long-term roots of structural inequalities-proliferation of child labor and child
prostitution?
A. Raise views of human right abuse.
B. Organize regular programs for information and discussion of
human rights.
C. Conduct skills training.
D.

Raise questions over the governments commitment to


rebuild human rights.

40. Which explains the reason why there are continuous and increasing human rights
violations?
A. The United Nations General Assembly approved only
resolutions on human rights and the basic freedoms which
are not binding
B. The solutions used are ineffective.
C. The United Nations as an International body is rather slow in
the exercise of its powers.
D.

The United Nations uses a single solution on all forms of


human rights violations.

41. Rivers in Metro Manila are the identified causes of heavy flood.
account this to
A. waste burning in home yards
B. waste dumping on river banks
C. waste classification
D. waste incineration

Authorities

42. Piggery farms affected by FMD (Foot and Mouth Disease) are prohibited from
A. castrating their piglets
B. isolating their piglets from sows
C. butchering their pigs for the market*
D. mating their sows when in heat
43. Mining in Mt. Diwalwal was stopped by DENR and the court.
reason?
A. Water was found polluted by mercury.
B. Water is reduced by mining.

What was the

C.
D.

Permits were not issued to miners.


Some miners became rich.

44. Earth day is celebrated on April 22 every year. Historically this day took off in the
Philippines in the
A. 1960s
B. 1950s
C. 1940s
D. 1970s
45. The man could not be convinced by the findings on the fallen bridge. To be sure
he advised verification through.
A. Objectivity
B. Critical analysis
C. Curiosity
D. Rationality
46. Wastes from the kitchen are considered useful if properly separated.
these classified?
A. A non-degradable
B.
C.
D.

How are

Biodegradable
Metallic recesses
Degradable

47. Beef from Europe was banned for some time.


contaminated meat is reported to affect
A. Brain function
B. Eye sight
C. Blood circulation
D. Muscular system

The disease caused by these

48. The professionals who handle animal disease are known as


A. Veterinarians
B. Doctors of cattle
C. Animal husbandry
D. Doctors of medicine
E.
49. The Philippine Air Force took off to scatter salt to the existing clouds. This was an
attempt to
A. Encourage earthquake occurrence
B. Produce rain over dry fields*
C.
D.

Drive the clouds further to the sea


Avoid water evaporation

50. Dengue affected patients are easily identified through


A. High fever with rashes in 2-3 days
B. Presence of mucous in the nostrils
C. Presence of red spots in the arms and other body parts
D. Presence of black spots around the eyes
51. The Bureau of Fisheries has discovered way to control the sex of different fish
varieties. They succeeded best in
A. Tilapia*
B. Carp
C. Talakitok
D. Milkfish or bangus
52. Agriculturists advise farmers during of El Nio to raise
A. More root crops
B. More corn crops
C. More rice crops
D. More leafy vegetables
53. There were people considered scientifically conscious and advanced. They were
A. Greeks and Chinese

B.
C.
D.

Egyptians and Greeks


Babylonians and Egyptians
Babylonians and Greeks

54. Dengue fever has caused death to many children and adults. All these are traced
to
A. Poor disposal of waste
B. Poor hoarding of water in tanks
C. Poor maintenance of home compost pits
D. Poor ventilation
55. Very lately the WHO gave a warning on the recurrence of ______________.
A. TB in Malaysia
B. Cholera in Asia *
C. Malaria in the Philippines
D. Anthrax in Mindanao
56. The public school children upon entering grade one were found suffering greatly
of:
A. Kidney trouble
B. Tuberculosis
C. Eye disease
D. Dental cavities*
57. During the bombing of the World Trade Center on September 11, 2001, the world
saw the events at once. This is accountable to
A. Light waves
B. Digital exposure
C. Technological communication
D. Radio waves
58. A huge rock atop a
does it possess?
A.
B.
C.
D.

hill is about to roll down. At this instance what kind of energy

59. Identify the space


landing at Kennedy
A.
B.
C.
D.

vehicle that exploded in the air some 10 minutes before


Space Center.
Mariner 1
Columbia
Apollo 11
Discovery

Mechanical
Potential
Instantaneous
Kinetic

60. Simple goiter is caused by a deficiency in the secretion of which hormone?


A. Pituitrine
B. Thyroxine*
C. Adrenaline
D. Estrogen
61. Which is the first law of thermodynamics?
A. Energy can be destroyed by other powerful forms.
B. Energy is dynamic in nature.
C.
D.

Energy is destroyed when different forms combine.


Energy cannot be created nor destroyed.

62. Boyles law states that at constant temperature volume of a gas is ______________.
A. inversely proportional to temperature
B. directly proportional to pressure
C. directly proportional to volume
D. inversely proportional to pressure
63. When ice cubes form vapor without passing through the liquid state, what process
takes place?

A.
B.
C.
D.

Transpiration
Sublimation
Condensation
Evaporation*

64. Boiling water was poured until it filled a pitcher and a cup. Which of the following
observations were recorded?
A. Water in both containers is of the same temperature.
B. Water in the cup has a lower temperature.
C. Water in the pitcher has a higher temperature.
D. Water in both containers released potential energy.
65. Trace the conversion of energy that occurs when dry cells are used in a flashlight.
A. Chemical energy to heat energy
B. Chemical energy to light energy *
C. Kinetic energy to potential energy
D. Chemical energy to kinetic energy
66. Ms. Ilandrez has a rare collection of orchids. She applied jelly on her orchids
leaves. Soon the orchid slowly wilted because
A. the leaves were shiny because of jelly
B. the stoma in the leaves were covered *
C. the jelly consumed the heat
D. the leaves were made hot by the jelly
67. The DENR are strict on giving permits to mining and other land use.
secured to be able to establish a factory?
A.
B.
C.
D.

What is

Electrogram
License to operate
License to cut trees
Environmental Control Compliance (ECC) *

68. Flood has devastated Mindanao and many provinces. The root cause is traced to
A. wanton destruction of gold mines
B. conversion of agricultural land to subdivisions
C. wanton cutting of forest trees *
D. using wood for fuel
69. The menace of AIDS does not exclude children. How are the mothers infected
with AIDS handled by the Department of Health? They are ______________.
A. given fertility pills
B. monitored closely in clinics *
C. made to marry young
D. ligated early
70. The Philippines has
A.
B.
C.
D.

lost its fish breeding places. The destruction is found most in


shell fish destruction
mangrove destruction
mountain side shores
deep sea areas

71. Fertilizers are encouraged in farming; however what is the best step before
applying any fertilizer? That is through
A. soil analysis
B. mineral content analysis
C. water content analysis
D. soil thickness measurement
72. The Health Department went on a zero campaign among babies and children from
3 months to 10 years. This was done to
A. stop the spread of polio*
B. reduce typhoid fever
C. reduce occurrence of cancer
D. counteract tuberculosis

73. There was a recent epidemic among cows in Europe.


preventive measures against mad cow through
A. vaccination of cows
B. banned importation of cows *
C. meat inspection
D. exporting beef to Europe

The Philippines took

74. Very recently several monkey farms were closed and their pets destroyed. The
monkeys were found infected with
A. blood disease
B. tetanus
C. AIDS
D. rabies
75. Fish farms abound for live fish brought to markets.
demand oxygen care?
A. crabs
B. bangus
C. sapsap
D. lapu-lapu*

Which of these variety

76. People are discouraged in burning their garbage. This has been destructive to
A.

neighbors health

B.
C.
D.

plants around
birds in the area
the ozone layer *

77. Several hog farms were affected by the foot and mouth disease.
entered the piggery cites are
A. made to use gowns and caps
B. pork are given fee
C. vaccinated without AMF
D. made to step on disinfectants *

Visitors who

78. Children are discouraged to play in flood water since


A. feces of cockroaches are floating
B. feces and urine of animals abound *
C. urine of rats are contagious
D. urine of people are around
79. During wars, what is most dreaded by nations?
A. biological war *
B. anthrax
C. atomic bomb
D. hydrogen bomb
80. Mothers sick with diabetes are usually found to be fond of
A. Candies *
B. crabs
C. tamarind
D. meat
81. The Philippines and other nations health are well ministered by an organization.
Which is mentioned for this?
A. UNESCO
B. World Health Organization *
C. United Arabs
D. United Nations
82. The Supreme Court approved a bidding to dispose of waste legally.
opposition was against it because of _________________.
A. burying waste
B. biodegradable segregation
C. incineration of waste

The

D.

separating plastic and paper

83. Chicken pox usually occurs in August. At what stage is it very contagious?
A. when wound are peeling off
B. when wounds have swollen
C. when all wounds have disappeared
D. during the first week

84. The school physician diagnosed children in inner towns suffering from night
blindness. He prescribed the taking in of
A. Vitamin C
B. Iron sulfate
C. Vitamin A
D. Vitamin B12
85. Cleaning of school premises is a part of school cleanliness. What is the best time
to conduct ground improvement?
A.
B.
C.
D.

7:15
3:30
5:10
9:20

A.M.
P.M.
P.M.
A.M.

86. Julias mother had German measles when she was carrying Julia in her womb.
Which of these physical appearances shows the effect on the child?
A. Blindness
B. Defective sight
C. Retardation in speech
D. Defective hearing
87. In order not to lose
A.
B.
C.
D.

the mothers teeth, what is encouraged in their diet?


Iron
Fat
Carbohydrates
Vitamins

88. During a class in gardening, one boy was bitten by a snake. What is the best first
aid?
A. Tie a band around the wound.
B. Wash the wound with soap and water.
C. Pound garlic and spread it over the wound.
D. Tie a band above the bite.
89. A child saw an incident of killing in school. The authorities could not compel the
child to go to court. What should be present to allow the child to testify?
A. Consent of the trying judge
B. Consent of the school principal
C. Consent of the classroom teacher
D. Consent of the parent to testify and participate
90. The government wants a piece of private land for a government project.
owner resisted the government. How may the government own the land?
A. Right of eminent domain
B. Right of government ownership
C. Right of habeas corpus
D. Right of sovereign over the governed

The

91. Which of the following processes is better associated with the technology than
with Science?
A.
B.
C.
D.

Constructing concepts and laws


Discovering forms of matter
Designing a workable system *
Observing natural phenomena

92. What term is used for one who depends entirely on others for subsistence?
A. Subordinate
B. Parasite *
C. Medium
D. Host
93. Electrical energy is used to power a light bulb. How does the amount of electrical
energy used compare to the amount of light energy produced?
A. depends on the size of the light bulb
B. same
C. more
D. less
E.
94. Any change in the base coding of a DNA molecule alters the trait associated with
the gene, hence the organism may appear with a trait that is totally different from
those of the parents. What is this change called?
A. Gene mutation
B. Gene meiosis
C. Trait linkage
D. Sex-linked inheritance
95. Which of the following is the primary reason why scientists repeat the
measurement they take during experiments?
A. To estimate on experimental error *
B. To change the experimental conditions
C. To list the data in a tabulation
D. To find out if the equipment is working
96. Which of the following practices can lessen soil erosion?
A. plowing at short distance in-between
B. constructing terraces
C. continuous farming
D. building canals
97. In order to be able to escape the Earths gravitational pull, an object must be
travelling at a speed of _____________.
A. 100, 000 km / hr
B. 400, 000 km / hr
C. 10, 000 km / hr
D. 40, 000 km / hr
98. Using a genetic engineering technique, researchers were able to produce a rare
protein which may be a possible cure for cancer. Which is it?
A. Thymine
B. Adenine
C.
D.

Interferon *
Mutagen

99. A weather disturbance accompanied by a 50-km / hr wind is classified as a


_________.
A. Tropical depression
B. Storm
C. Typhoon
D. Tornado
100.

Who is the scientist who is likewise recognized as an International Statesman?


A. Benjamin Franklin *
B. Thomas Jefferson
C. Alexander Hamilton
D. George Washington

101.

What nuclear process occurs in the explosion of an atomic bomb?


A. Disintegration
B. Integration
C. Fission *

D.

Fusion

102.

What is the cell


A.
B.
C.
D.

wall of plants made of?


Plastids
Cytoplasm
Cellulose *
Lipids

103.

Which type of a
A.
B.
C.
D.

change results in the formation of a new substance?


Potential
Gradational
Chemical *
Physical

104.

In which process does a chemical change take place?


A. Freezing water into ice cubes
B. Boiling water into steam
C. Condensing clouds into rain
D. Manufacturing starch in leaves *

105.

Which modified
A.
B.
C.
D.

106.

Which one shows that we form part of the carbon cycle?


A. During oxidation, burning objects release oxygen.
B. We take in carbon dioxide and oxygen at the same time.
C.
D.

stem is used for the storage of food?


Stem runner
Tomato
Potato
Grass stem

During respiration, we take in oxygen and release carbon


dioxide.*
We take in oxygen and release it during respiration.

107. Which of the following is used by medical workers in obtaining information


about internal body parts?
A. Ultrasound
B. Infrasound *
C. Laser
D. Scan
108.

Where do we trace the El Nio phenomena?


A. rising ocean levels
B. very severe earthquakes
C. annual changes in season
D. depletion of the ozone layer *

109.

Which of the following materials can transmit sound best?


A. gas
B. metal
C. water
D. air

110.

What is the best way to conserve soil in a hill?


A. plow the hill regularly
B. build terraces *
C. construct fences
D. irrigate the area

111.

Which instrument can measure the intensity of earthquakes?


A. anemometer
B. seismograph *
C. barograph
D. psychrometer

112.

Why are rainbows formed?


A. raindrops are composed of 7 colors
B. sunlight is absorbed by raindrops
C. raindrops act as prisms *
D. sunlight acts as prism

113. Which part of the cell determines the physical characteristics that could be
inherited by an offspring?
A. mitochondria
B. gene *
C. chromosome
D. centrosome
114.

Why do mountain climbers use oxygen equipment at high altitudes?


A. There is less oxygen in upper atmosphere. *
B.
C.
D.

There is no air at high altitudes.


Oxygen fails to join the atmosphere.
Oxygen unites with nitrogen in the atmosphere.

115.

The application
A.
B.
C.
D.

of scientific concepts and principles is called __________.


Ecology
Physiology
Technology *
Biology

116.

Why do we feel
A.
B.
C.
D.

cool after perspiring?


The perspiration was condensed.
Air entered the pores of the skin.
The perspiration has evaporated.
The perspiration was absorbed.

117.

Choose an example of a physical change in matter.


A. paper torn to pieces *
B. acid rain formed
C. milk turned sour
D. milk became cheese

118. Why does greenhouse effect result to warmer temperature near the Earths
surface?
A. Ozone traps ultraviolet radiation.
B. Carbon dioxide traps infrared radiation.
C. Clouds trap infrared radiation.
D. Soil absorbs incoming radiation.
119. How can you help minimize the heavy pollution that is caused by excessive
carbon dioxide emissions in the surroundings?
A. Burn garbage in your own backyard.
B. Dispose garbage through incineration.
C. Leave the garbage to rot in open space.
D. Bury garbage rather than burn it.
120.

In which medium would sound travel faster?


A. copper
B. water
C. oxygen
D. air

121. Who conducted extensive experiments on pea plants, formulated the laws of
heredity and is considered as the Father of Genetics?
A. C. Barr
B. Gregor Mendel *
C. J. Lamark
D. Charles Darwin

122.

Environmental pollution is classified as violence that is termed ____________.


A. direct
B. ecological *
C. socio-cultural
D. structural

123.

Rabies comes from dog and other bites. How could this be prevented?
A. Have the dog vaccinated with anti-rabies. *
B. Tie the dogs at all times.
C. Keep the dogs in cage.
D. Kill the dog that bites.

124.

How do oil wastes on the surface of a river affect the organisms in it?
A. become sticky
B. accumulate other wastes
C. block sunlight *
D. absorbed by them

125.

Lack of appetite
A.
B.
C.
D.

and less sleep gave her a __________ appearance.


lanky
haggard *
easy-going
superstitious

126. A clear image can be formed on a still water surface. Which principle of light
can explain it?
A. The angle of reflection is equal to the angle of refraction.
B. Both angles produce irregular reflection.
C. The angle of incidence is equal to the angle of reflection.
D. The angle of refraction is equal to the angle of incidence.
127.

Light insects can walk on water surface. Why is it possible?


A. due to surface tension *
B. water surface is denser
C. due to surface adhesion
D. due to surface capillarity

128.

Which instrument is used to measure the intensity of earthquake?


A. Barograph
B. Thermograph
C. Seismograph *
D. Hygroraph

129.

The Filipino family believed that tuberculosis is due to


A. poor hygiene in coughing and spitting
B. spitting everywhere by active TB patients
C. lack of Vitamin A
D. lack of Vitamin C

130. Children in the mountain ranges are diagnosed as suffering from goiter.
Determine the cause of such disease.
A. lack of iodine in food *
B. excess of iodine in diet
C. excess of vitamins in diet
D. lack of iron in diet
131.

One good rule to avoid intestinal disease is __________________.


A. wash your hands before eating *
B. wash your feet before going to bed
C. sleep under a mosquito net
D. avoid insect bites

132.
I.
II.
III.
IV.

Which should be TRUE if Earths rotation axis did not tilt?


Days and nights would be the same length everywhere on Earth.
There would be no hours of darkness on points along the equator.
Earth would have no seasons.
Each part of the Earth would have the same daily temperature pattern.
A.
B.
C.
D.

II and IV
III and IV
IV and I
I and III

133. Tides, caused by the moons gravity, create a frictional force that is gradually
slowing down Earths rotation speed. One million years from now, scientist may
discover that compared to today. Earths
A. day is longer
B. year is shorter
C. day is shorter
D. year is longer
134.

When do doctors declare dengue fever as an epidemic?


A. When two are hospitalized from the neighborhood.
B. When the proportion of patients reaches at least 15% of the
population.
C. When every household of fine houses is sick with high fever.
D. When everyone is suffering from a cold.

135. Opening of cement plants require getting a DENR clearance. Which of these
clearances are required?
A. Air Pollution Clearance
B. Waste Disposal
C. BIR
D. Environmental Clearance (ECC) *
136. The latest discovery in animal industry is in biological science. Where was this
mostly used?
A. In artificial insemination *
B.
C.
D.

In tissue transplant
In artificial breeding of cattle
In cloning of cows

137. Health centers conduct home visitation. Babies are vaccinated early to avoid
baby tuberculosis. However, what is the WHO findings?
A. Most fathers spit blood and infect babies.
B. Babies are immune from TB
C. Most children suffer from pulmonary complex
D. Mothers sick with TB transmit their disease by breast feeding
138.

A recent cause
A.
B.
C.
D.

of teachers illness is traced to:


urinary tract infection
anemia
tuberculosis *
tonsillitis

139.

One approach to lessen vehicular air pollution is:


A. revenue twenty or more year old vehicles in the street
B. ban diesel-using cars on the highway
C. ask pedestrians to use bicycles
D. utilizing the MRT and LRT

140.

Which surface heats up and cools fast?


A. Land *

B.
C.
D.

Water
Vegetation
Sea

141. A certain object has a mass of 250 kg. When it is brought to outer space,
what change is observed with its mass?
A. Increased in size
B. Decreased in amount
C. Increased in weight
D. Remained the same
142.

Why do we see
A.
B.
C.
D.

the sun rise in the east?


The Earth rotates clockwise.
The Earth rotates counterclockwise.
The sun is located in the East.
The Earth revolves towards the East.

143.

Which of the following decibels would likely cause noise pollution?


A. 30-40
B. 90-100
C. 60-70
D. 10-15

144.

Why are animals described as warm-blooded organisms?


A. Body temperature changes with the environment.
B. Temperature goes beyond 27 degrees Celsius.
C. Body temperature does not change. *
D. Temperature does not drop below 27 degrees Celsius.

145.

Which term indicates the amount of space occupied by matter?


A. Density
B. Weight
C. Volume
D. Mass

146.

Why do the four


A.
B.
C.
D.

147.

How many protons, neutrons, and electrons does Uranium 238 have?
A. 146 electrons, 92 protons, 146 neutrons
B. 146 protons, 92 electrons, 92 neutrons
C. 92 protons, 92 neutrons, 146 electrons
D. 92 protons, 146 neutrons, 92 electrons

148.

Choose the 6-carbon simple sugar.


A. sucrose
B. maltose
C. lactose
D. glucose *

149.

Organisms that
A.
B.
C.
D.
E.

eat plants only are called _____________.


oviparous
herbivores *
omnivores
carnivores

150.

Which animal is
A.
B.
C.
D.

classified as herbivore?
Frog
Dog
Carabao *
Tiger

seasons occur on Earth?


The suns axis tilted.
The Earth rotates from west to east.
The Earth rotates on its axis.
The Earths axis is tilted. *

151. Which technique is employed by medical practitioners in examining some


internal organs usually in the abdominal region?
A. Ultrascan
B. Laser
C. Endoscopy
D. Infrasound
152. Which property of sound enables us to distinguish between a particular pitch
produced by a piano and that by a violin?
A. Intensity
B.
C.
D.

Frequency
Quality
Wavelength

153. Which is a new X-ray technique used to detect tumors and other minute
abnormalities?
A. Beam scanner
B. Radiation scanner
C. Laser
D. Micro-detector
154. What type of thermonuclear reaction occurs in the sun which
tremendous amount of heat energy?
A. Fission
B. Radiation
C. Convection
D. Fusion *
155.

Which organism
A.
B.
C.
D.

generates a

receives the least amount of energy in the energy pyramid?


Snake
Hawk *
Chick
Corn plant

156. When we think, study and react too consciously, which part of the brain
functions?
A. Cortex
B. Cerebellum
C. Cerebrum
D. Thalamus
157.

Which is NOT an abiotic component of an ecosystem?


A. Sulfur
B. Carbon
C. Temperature
D. Organism *

158.

Which of the following organisms transmit the malarial parasites?


A. Ants
B. Flies
C. Culex mosquitoes
D. Anopheles mosquitoes

159.

Which type of rock does granite belong?


A.
B.
C.
D.

160.

Metamorphic
Deposits
Igneous
Sedimentary

Why is Uranium classified as a radioactive element?


A. It is artificially produced in the laboratory.
B. It emits alpha particles.

C.
D.

It is a useful radio component.


It is observed to be hyperactive.

161. Food scraps and food peelings can be converted to humus through which of
the following disposal system?
A. Open dump
B. Sanitary landfill
C. Recycling
D. Composting
162. The astronauts
surface. Why?
A.
B.
C.
D.

cannot communicate with one another while on the moons


Temperature shuts down sound
Air waves are irregular
There is no weather that occurs
There is no atmosphere *

163. Why is velocity classified as a vector quantity?


Because velocity is
______________.
A. speed expressed in kilometers per hour
B. force and motion
C. rate of motion
D. speed and direction
164. What process is responsible for the transporting of broken pieces of rocks from
where they are formed to other places?
A. Deposition
B. Weathering
C. Erosion *
D. Gravitation
165.

What may occur when the blood vessel going to the brain is blocked?
A. Heart attack
B. Hemorrhage
C. Stroke
D. Tumor

166.

Which gas is released by plants during photosynthesis?


A. Carbon dioxide
B. Oxygen *
C. Hydrogen
D. Nitrogen

167. Which is the simplest substance which cannot be decomposed further by


ordinary chemical means?
A. Molecule
B. Compound
C. Mixture
D. Element *

168.

Glass breaks easily. Which property of matter is exhibited?


A. Tenacity
B. Malleability
C. Ductility
D. Brittleness

169. What phenomenon occurs when the length of day and night varies depending
on the inclination of the Earths axis?
A. Coriolis effect
B. Spring
C. Solstice
D. Equinox
170.

Which of the following factors contribute to the greenhouse effect?

A.
B.
C.
D.

Excessive use of solar energy


Destruction of our forests
Use of carbon dioxide by green plants
Depletion of minerals by over farming

171.

What does a speedometer of a moving car show?


A. Average acceleration
B. Instantaneous speed
C. Average speed
D. Instantaneous acceleration

172.

Why is plastic commonly used as handle of kitchen utensils


A. It is lighter than metal.
B. It is a good conductor of heat.
C. It is a poor conductor of heat. *
D. It is denser than metal.

LET Final Coaching in Professional Education


Directions: Read carefully each item. Then encircle the letter of the choice that
describes correctly what ever is needed in each item.
PART I. (Time Frame: 45 minutes)
Topic: Sociological, Anthropological, Historical, Philosophical, and Ethical
Foundations of Education including Philippine Educational System
1. Who perpetuated the idea that reason is superior to experience as a source
of knowledge thus schools must develop human reason?
a. Rationalists
c.
Hedonists
b. Humanists
d.
Stoics
2. We encounter people whose prayer goes like this: O God, if there is a God;
save my soul, if I have soul. From whom is this prayer?
a. Stoic
c.
Agnostic
b. Skeptic
d.
Empiricist
3.

The commonly-used question and answer method that includes debate and
discussion is in consonance with the method of
a. Plato
c.
Aristotle
b. Socrates
d.
Confucius

4. From whom did we get the taxonomical practice of arranging all species in
their natural order in accordance with their complexities of form, the main
levels being the inanimate, the vegetative, the animal and the rational?
a. Pythagoras
c.
Watson
b. Plato
d.
Aristotle
5. Whose influence is the education program that puts emphasis on selfdevelopment through the classics, music, and rituals?
a. Mohammed
c.
Lao Tsu
b. Confucius
d.
Buddha
6. From whose teaching is the study and passing on of tradition emphasized?
a. Lao Tsus
c.
Mohammeds
b. Buddhas
d.
Confuciuss

7. If given the power, you ought to govern your subjects with moral power. From
whom is this advice?
a. Lao-Tsu
c.
Mencius
b. Mao-tse-tsung
d.
Kung-fu-tzu
8. What you do not want to do to yourself, do not do to others. From whom
does this advice come from?
a. Christian and Confucianist
c.
Hindu and Christian
b. Muslim and Christian
d.
Christian and Buddhist
9.

Both Muslims and Christian value marriage but the Muslim practices
polygamous marriage while the Christian practices monogamous marriage.
What is it called?
a. Acculturation
c.
Ethical relativism
b. Enculturation
d.
Cultural relativism

10. The Second World War has caused too much destruction to life and
property and the Allied Forces consisting of the United States, Great Britain
and France wanted to end it, but Japan adamantly refused. To force Japans
surrender, the Allied Forces dropped the first atomic bomb on Hiroshima
and Nagasaki on August 9, 1945. Which basic principle of morality applies
to this case?
a. Always do what is right.
b. Principle of double effect
c. The end does not justify the means.
d. Between the two evils, do lesser evil.
11. There are many instances in the classroom that point to cultural alienation.
In what way does cultural alienation fail to contribute to nation-building?
a. The Filipino has no sense of pride.
b. The Filipino closes himself to foreign influence
c. The Filipino operates on a Filipino model of development.
d. The Filipino will look at his culture as something superior to others.
12. The law of karma, what you sow you reap, are words often said when
Filipino attempt to explain a misfortune perceived to be rightly deserved by
someone. This originated from the _____.
a. Hindus
c.
Hebrews
b. Greeks
d.
Chinese
13. What concept of education makes every individual a useful citizen to the
state?
a. Humanitarianism
c.
Conformity
b. Utilitarianism
d.
Security
14. The area of philosophy which deals on the nature of knowledge and the
best method of teaching is
a. Ethics
c.
Metaphysics
b. Aesthetics
d.
Epistemology
15. He introduced the kindergarten or childs garden to the educational
system.
a. Johann Herbart
c.
Maria Montessor
b. Friedrich Froebel
d.
Edward Paparazzii
16. Teacher Ol subscribes to the mental discipline approach. Which is she
convinced of?
a. The more difficult the subject, the more the student exercises his

mind, the greater the value of the subject.


b. The less the student engages himself in the mental exercise, the
better for the student.
c. The easier the subject, the greater the value of the student.
d. The more the bodily exercises, the better for the student.
17. Who is the most popular among the pragmatists?
a. Jean Piaget
c.
William James
b. John Dewey
d.
Amos Comenius
18. As a naturalist, Jacques Rousseau claimed that everyone is essentially
good. This view can help the teacher best when ______.
a. dealing with misbehaving students
b. helping the fast learners in class
c. teaching the students some values
d. knowing the students potentials
19. Teacher U teaches to his pupils that pleasure is not the highest good.
Teachers teaching is against what philosophy?
a. Empiricism
c.
Hedonism
b. Epicureanism
d.
Realism
20. Teacher A is convinced that she should teach her students to appreciate
themselves for who they are and to accept the responsibility for their
thoughts, feeling and actions. Which philosophy governs Teacher As
thought?
a. Existentialism
c.
Perennialism
b. Realism
d.
Naturalism
21. The emphasis on the scientific method is backed up by which philosophy?
a. Idealism
c.
Utilitarianism
b. Empiricism
d.
Perennialism
22. On which philosophy is the inclusion of the study of Rizal and the other
national heroes in the school curriculum based?
a. Existentialism
c.
Idealism
b. Pragmatism
d.
Realism
23. You regard the classroom as a small democracy where you help your
children live the democratic way of life. To which educational philosophy do
you adhere?
a. Reconstructionism
c.
Existentialism
b. Progressivism
d.
Realism
24. The student-centered curriculum belongs to which group of educators?
a. Reconstructionists, perennialists
b. Progressivists, reconstructionists
c. Progressivists, essentialists
d. Perennialists, essentialists
25. Principal B tells her teachers that training in the humanities is most
important. To which educational philosophy does he adhere?
a. Existentialism
c.
Essentialism
b. Perennialism
d.
Progressivism
26. What recent education development relates with perennialism?
a. Exercising the students higher order thinking skills (HOTS)

b. Suiting the strategies to the students to the students learning styles.


c. Individualizing the learning experience and activities
d. Having drills and route activities that develop learning habits.
27. Which does the empiricist emphasize?
a. Knowledge arises from the mind
b. Information is interpreted by the mind
c. Experience is the only source of knowledge
d. Knowledge is derived from reason without the aid of the senses
28. Which groups of philosophers have opposite views?
a. Rationalists and Idealists
b. Rationalists and Naturalists
c. Rationalists and Empiricists
d. Rationalists and Existentialists
29. After being humiliated by his teacher, student B evaluates that teacher very
poorly despite teachers excellent performance. Which trait is illustrated by
the student Bs behavior?
a. Rationalism
c.
Impersonalism
b. Personalism
d.
Particularism
30. Which of the following prepositions is attributed to Plato?
a. Human beings create their own truths.
b. Truth is relative to a particular time and place.
c. Sense perception is the most accurate guide to knowledge.
d. Learning is the discovery of truth as latent ideas are brought to
consciousness.
31. Matthew Arnolds ideal person was the person whose powers were all in
balance. The following enumerate which have to be developed for a person
to become Arnolds ideal person, EXCEPT:
a. brain, mind, reason
b. head, heart, hands, health
c. knowledge, skills, attitudes
d. cognitive, psychomotor, and affective powers

32. Teacher L is a typical idealist teacher. Which belief about knowledge is he


likely to advocate?
a. Knowledge through reasoning
b. Knowledge through trial and error
c. Knowledge through hypothesis testing
d. Knowledge from direct, concrete experience
33.

In a study conducted, the pupils were asked which nationality they


preferred if given a choice. Majority of the pupils wanted to be Americans.
In this case, in which obligation relative to the state are schools seemed to
be failing? In their obligation to
a. Promote obedience to the laws of the state.
b. Respect for all duly constituted authorities
c. Instill allegiance to the Constitution.
d. Promote national pride

34. Which statement on responsibility is CORRECT


a. A persons degree of responsibility is fixed at birth
b. A persons sense of responsibility cannot be developed
c. A persons sense of responsibility decreases as he grows
d. A persons sense of responsibility increases as he grows in age
35. As a teacher, you are a rationalist. Which among these will be your guiding
principle?
a. I must teach the child every knowledge, skill, and value that he needs
for a better future.
b. I must teach the child that we can never have real knowledge of
anything.
c. I must teach the child to develop his mental powers to the full.
d. I must teach the child so he is assured of heaven.
36. Referring on the characteristic of a progressivism curriculum, which does
NOT belong to the group?
a. Flexible
c.
Child-centered
b. Relevant
d.
Subject-mattered
37. Which is the weakness of the Montessori approach?
a. Fosters independence
b. Emphasizes verbal interaction
c. Neglects childrens social interaction
d. Fosters development of cognitive skills
38. According to the existentialist, every person is in the same predicament and
has the same possibilities. What does this imply?
a. Every person must go through the same form of education
b. Every person must have access to education
c. Every person must choose to go to college
d. Every person must go to college
39. Which, according to Jocano, is a strength of the Filipino people?
a. Sense of commitment to nationalistic ideals
b. Sense of historical or national identity
c. Sense of national pride
d. Sense of humor
40. Teacher Flor and teacher Bea are rivals for promotion. To gain the favor of
the promotional staff, Teacher Bea offered her beach resort for free for the
members of the promotional staff before the ranking. As one of the
contenders for promotion, is this becoming of her to do?

a. Yes, the rare invitation will certainly be welcomed by an overworked


promotional staff
b. Yes, this will be professional growth for the professional staff
c. Yes, theres nothing wrong with sharing ones blessings
d. No, this may exert undue influence on the members of the
promotional staff and so many fail to promote someone on the basis
of merit
41. In the preamble of the Code of Ethics of Professional Teachers, which is
NOT mentioned about teachers?
a. With high moral values as well as technical and professional
competence
b. Possess dignity and reputation
c. Duly licensed professional
d. LET passers
42. A teacher discovers that a product of a certain bottling company brings
about damage to teeth. Much as he wants to share the products of his
research, he could not because of harassment from all sides. Which
teachers right is violated?
a. Right to make a livelihood
c. Academic freedom
b. Right to ones honor
d. Right to property
43.. A teacher CANNOT unduly trample upon or suppress a students right
without moral guilt because rights are ________.
a. perfect
b. inalienable
c. inviolable
d. adventitious
44. Which is unethical for teachers to do?
a. Maintaining cordial relationship with parents
b. Obeying the legitimate policies of the school administration
c. Conferring with the next of kin about the problems and needs of
students
d. Refusing to serve in worthwhile neighborhood activities as these will
adversely affect teaching
45. Which is an example of perfect duty?
a. Supporting a poor but deserving student to school
b. Paying the worker the wages agreed upon
c. Donating an amount for a noble project
d. Giving alms to the needy
46. What kind of social environment prevails in your class if you, as the teacher,
are the primary provider of information, opinions, and instructions?
a. In between democratic and Laissez faire
b. Laissez faire
c. Authoritarian
d. Democratic
47. A teachers quarrel with a parent makes her develop a feeling of prejudice
against the parents child. The teachers unfavorable treatment of the child
is an influence of what Filipino trait?
a. Lack of self-reflection
b. Extreme family-centeredness
c. Sakop-mentality

d. Extreme personalism
48. When a delinquent, failing student comes to you for help and asks you for
extra work so he can cope, what is ethical for you to do?
a. Give him the extra work he is asking, and then if he passes quality
work, give him a passing mark.
b. Make the extra work that he is asking extremely difficult for him to
learn a lesson.
c. Extend the needed assistance in solving the students difficulties, not
necessarily by giving the extra work that he is asking.
d. Give an easy extra work so he can really be helped to pass.

49. A teacher discovers that a product of a certain bottling company brings


about damage to teeth. Much as he wants to share the products of his
research, he could not because of the harassment all sides. Which right is
violated?
a. Right to make a livelihood
b. Right to ones honor
c. Academic freedom
d. Right to property
50. Teacher A, a young high school teacher, senses that the oldest student in
class, Student Z, is attracted to her. The attraction is becoming mutual. Is
there anything unethical?
I. None, unless Teacher A gives preferential treatment to Student Z.
II. None, there is nothing wrong with being attracted to someone. It is what
we do with it that makes it ethical or unethical.
III. There is. The attraction is the beginning of unethical action.
a. III
b. I and III
c. II and III
d. I and II
51. Teacher A put together the output of her colleagues in one workshop and
published it with her name as author. Which is unprofessional about Teacher
As behavior?
a. Giving due credit to others for their work.
b. Not giving due credit to others for their work
c. Failing to correct what appears to be unprofessional conduct
d. Holding inviolate all confidential information concerning associates
52. In what way can teachers uphold the highest possible standards of the
teaching profession?
a. By working for promotions in the system
b. By continuously improving themselves personally and professionally
c. By pointing out the advantages of joining the teaching profession
d. By good grooming to change peoples poor perception of teachers
53. Teacher Es sideline is selling Encyclopedias for Children on weekends.
Her usual clientele are the parents from the parents of pupils who are on the
verge of failing or those vying for the Top Ten. In fact, she always
succeeded in selling encyclopedia to these parents. Is there unethical with
the way Teacher E does her sideline?
a. Yes, there is. She may become less objective in her evaluation of the
work of these pupils concerned.
b. Teachers are supposed to promote reading. No, there isnt. A
teachers salary is meager and selling books is respectable way of
augmenting Teacher Es income.
c. No, there isnt. Books and teachers mix.
d. Set aside the study of local history.
54. Each teacher is said to be trustee of the cultural and educational heritage of
the nation and is obliged to transmit to learners such heritage. Which
practice helps fulfill such obligation?
a. Observe continuing professional education to be the best Filipino
teacher.
b. Study the life of Filipino heroes and pass on their deeds worth

emulating to students
c. Supplant indigenous culture with foreign culture claimed to be
superior.
d. Teach the latest instructional technology to Filipino children.

55. Who is NOT covered by the Code of Ethics of Professional Teachers?


a. Teachers in all educational institutions at the preschool, elementary,
and secondary levels
b. Teachers of academic, vocational, special, technical, or non-formal
institutions
c. All full time or part time public and private school
d. Teachers in the tertiary level
56. Which attitude is exemplified by a Boy Scout who says Bahala na! and dives
into a pool to save a drowning boy?
a. a daring attitude
b. A segurista attitude
c. An I dont care attitude
d. A daring attitude combined with pagmamalasakit
57.

Annual medical check up required of teachers is done in the interest of


a. Parents
b. School administrators
c. Filipino medical doctors
d. The state of every teacher.

58. According to Confucius, what is the best way to rule a people and attain
harmony?
a. By allowing people to do as they please
b. By consulting the governor
c. By moral example
d. By force
59. I get P5,000 pesos from the funds raised during Valentines Day without the
knowledge of the committee and donate it to a foundation serving a group of
handicap. Is my act of getting P5,000 from the funds justified?
a. Yes, the end justifies the means.
b. Yes, the means justifies the end.
c. No, the end does not justify the means.
d. Yes, if you intend to tell the committee afterwards.
60. Theft of school equipment like tv, computer, etc. by teenagers in the
community itself is becoming a common phenomenon. What does this
incident signify?
a. Communitys lack of sense of co-ownership
b. Inability of school to hire security guards.
c. Prevalence of poverty in the community.
d. Deprivation of Filipino schools.
61. If you agree with Rizal on how you can contribute to our nations
redemption, which should you work for?
a. Upgrading the quality of the Filipino through education
b. Opening our doors to foreign influence
c. Stabilizing the political situation
d. Gaining economic recovery.
62. Rights and duties are correlative. This means that
a. rights and duties arise from natural law.
b. rights and duties ultimately come from God.
c. rights and duties regulate the relationship of men in society.
d. each right carries with it one or several corresponding duties.
63. Under no circumstance shall a teacher prejudiced discriminatory against

any learner, says the code of Ethics. When is a teacher prejudiced against
any learner?
a. When he makes a nearsighted pupil sit in front
b. When he makes a farsighted pupil sit at the back
c. When he refuses a pupil with slight physical disability in class
d. When he considers multiple intelligence in the choice of his teaching
strategies.
64. In the faculty room, every one is talking about Teacher W who is tutoring for
a fee her own pupil who is running for honors. What is a professional thing
for the other teachers to do?
a. As a group report her to the principal
b. Correct her and remind her tutoring ones own pupil for a fee is
unethical.
c. Leave her alone; she might accuse you of meddling in her personal
life.
d. Talk to the presence of the tutee. Tell them what Teacher W is doing
is unprofessional.
65. What does conservative a Filipino student experience when she migrates to
the United States and witness for herself public display of affection?
a. Culture shock
c.
Enculturation
b. Acculturation
d.
Colonial Mentality
66. Which best explains what mainstreaming in the classroom means?
a. Teachers in SPED teachers are now teaching in the regular
classrooms.
b. Pupils in the regular classroom are now accepted in Special
Education centers.
c. Teachers of regular classes are now teaching children with disabilities
in SPED centers.
d. Principals who were previously enrolled in special education classes
are integrated into regular classes.
67.. This embodies the teachers duties and responsibilities and proper
behavior in performing them
a. Bill of Rights
b. Code of Ethics for Teacher
c. Philippine Constitution of 1987
d. Magna Carta for Public School Teacher
68. A melody may sound sad to one and yet when the individual notes are
played separately, there is nothing sad about it. The viewpoint is based on
the doctrine that
a. the whole experience is less than the sum of the parts
b. the whole experience is equal to the sum of the parts
c. the whole experience is more than the sum of the parts
d. the whole experience is not in any way related to the sum of the parts
69. A teacher by reason of having gotten tired of elections did not exercise her
right of suffrage last May 14, 2001. Which provision in the Code of Ethics
did she fail to observe?
a. Every teacher shall enjoy academic freedom.
b. Every teacher shall possess and actualize full commitment and
devotion to duty.

c. Every teacher shall vote and shall exercise all other constitutional
rights and responsibilities.
d. A teacher shall not engage in the promotion of any political, religious,
or other partisan interest
70.

Just as selected Filipino teachers today are sent abroad to study, the
Americans did the same in 1903. These teacher scholars were known as
______.
a. reformists
c.
insulares
b. pensionados
d.
peninsulares

71.

To ensure that all Filipino children are functionally literate, which


mechanism is meant to reach out to children who are far from a school?
a. Multi-grade classroom
b. A school in every barangay
c. Mobile teacher
d. Sineskela

72. Which program is DepEds vehicle in mobilizing support from the private
and non government sectors to support programs based on DepEds menu
of assistance packages?
a. Child-Friendly-School System
b. Every Child A Reader program
c. Adopt-A-School-Program
d. Brigada Eskwela
73. Which DepEd test do out-of-school youths and adults take to enable them to
enroll in college if they pass it?
a. National Achievement Test
b. National Secondary Achievement Test
c. National College Entrance Examination
d. Accreditation and Equivalency Test
74.

Which is the Teachers Professionalism Act?


a. R.A. 4670
c.
R.A. 7836
b. R.A. 7727
d.
R.A. 9293

75. How is the latest curriculum introduced in the elementary secondary schools
called?
a. Millenium Curriculum
b. 2002 Basic Education Curriculum
c. The Resructured Basic Education Curriculum
d. 2002 Elementary and Secondary Education Curricula
PAUSE FOR THE CHECKING AND ANALYSIS OF ITEMS

PART II. (Time Frame: 45 minutes)


Topic: Human Growth, Learning and Development
1.

What I hear, I forget. What I see, I remember. What I do, I understand.


What does this mean? Pupils learn best when they _______.
a. Work with groups
b. Learn independently
c Pay full attention in class
d. Take active part in the learning exercises

2.

In which way does heredity affect the development of the learner?


a. By placing limits beyond which the learner cannot develop
b. By compensating for what environment fails to develop
c. By making acquired traits hereditary
d. By providing equal potential to all

3.

Which of the following is NOT a developmental principle?


a. Social expectations mark every developmental period.
b. Development follows as orderly, predictable sequence.
c. Numerous studies have revealed how individuals develop.
d. Early development is more critical than later development.

4. John Watson said: Men are built not born. What does this statement point
to?
a. The effect of environmental stimulation on a persons development.
b. The absence of genetic influence on a persons development.
c. The ineffectiveness of training on a persons development.
d. The effect of heredity.
5. Neal is a boy who thinks that he is a good-for-nothing person. Evidently, he
does his school work poorly and his parents seem to have affirmed this
attitude at home. How can you effectively change this Pygmalion effect
(self-fulfilling prophecy)?
a. Tell Neals parents that their son needs help in school.
b. Assess Neals strengths and tap them naturally in class.
c. Assign Neal to be the leader during important activities.
d. Let Neal do something easy so that he will experience success.
6. Which teachers attitude best reflects his/her understanding of development
as a product of maturation and learning?
a. Fairness when giving grades and school marks
b. Patience when dealing with the slower ones
c. Creativity with the classroom strategies or task
d. Cheerfulness and enthusiasm when discussing.
7. Florence helps pupils determine the pronunciation and meanings of words by
analyzing roots, affixes, and derived forms. The process is called
a. Halo effect
c.
Pygmalion effect
b. Ripple effect
d.
Hawthrone effect
8.

Which conception of age is wrong?


a. Psychological age and social age are two different things.
b. Chronological age is the same as biological and psychological age.

c. Chronological age, biological, psychological and social age are


related.
d. Chronological age is different from biological and psychological age.
9. In what developmental stage is growth most rapid?
a. Infancy
c.
Early childhood
b. Adolescence
d.
Middle childhood
10. It is not wise to laugh a two-year old child when he utters bad word because
in his stage he is learning to _____________.
a. distinguish right from wrong
b. distinguish sex differences
c. consider others views
d. socialize
11. The developmental task of learning to get along with age mates belongs to
which developmental stage, according to Havighurst?
a. Early childhood
c. Adolescence
b. Early adulthood
d. Late childhood
12. All of the following describe the development of children aged eleven to
thirteen EXCEPT _____.
a. they shift from impulsivity to adaptive ability
b. they show abstract thinking and judgment
c. they exhibit increased objectivity in thinking
d. sex differences in IQ becomes more evident
13. Cognitive is to knowledge as affective is to ____.
a. Skills
d. Rights
b. Values
c. Obligations
14. Cognitive psychologists regard learner as one who comes into the
classroom ______.
a. knowing nothing
c.
uninterested to learn
b. full of experiences
d.
ignorant of schooling
15. Which term refers to the act of educating a child with special education
needs partially in a special education classroom and partially in a regular
classroom?
a. Individualization
c.
Inclusion
b. Mainstreaming
d.
Head start
16. A pupil has developed a love for reading keeps in reading for his enjoyment.
His motivation for reading is
a. both intrinsic and extrinsic
c.
extrinsic
b. insufficient
d.
intrinsic
17. Arnel has had difficulty getting the right solution to a problem in algebra.
Suddenly he saw how to solve the problem. Which of the following
explains this situation?
a. Revelation
c. Memory
b. Retention
d. Insight
18. Teacher S connects the new to one just completed to enable the students to
gain a holistic view of the subject. What psychological principle is invoked?
a. Stimulation
c. Apperception

b. Recognition

d. Conceptualization

19. If a child is bitten by a large, black dog, the child may fear not only black dog
but also other large dogs. Which conditioning process is illustrated?
a. Generalization
d. Acquisition
b. Discrimination
c. Extinction
20. Lilian was first asked to compare identical amounts of liquids in two short
glasses, The liquid from one of the two short glasses was poured into a
taller, slimmer glass of the same capacity. Lilian indicated that the amounts
of liquid in the two different glasses are still the same. What is Lilian a
capable of?
a. Assimilation
c.
Conservation
b. Accommodation
d.
Reversibility
21. Soc exhibits fear response to freely roaming dogs but does not show fear
when a dog is on leash or confined to a pen. Which conditioning process is
illustrated?
a. Extinction
c.
Generalization
b. Acquisition
d.
Discrimination
22. To make sense of our world we organize our experiences. We also change
our thinking to include new ideas and additional information. This process of
changing our thinking as a result of new ideas is termed by Piaget as
______________.
a. adaptation
c.
assimilation
b. construction
d.
adaptation
23. I am reminded of my Grade I teacher every time I see Miss Valdez because
their mannerisms are alike. Which principles of association explain this?
a. Contrast
c.
Frequency
b. Similarity
d.
Contiguity
24. I cannot forget my friends birthday for it comes one day after my birthday.
Which principle of association as applied to memory explains this?
a. context
c.
contiguity
b. frequency
d.
similarity
25. I cannot help but recall the Sisters convent which served as my boarding
house in high school now that I am in a noisy boarding house. Which
principle of association explains this?
a. contrast
c.
contiguity
b. frequency
d.
similarity
26. In which cognitive development stage is a childs unable to distinguish
between own perspective and someone elses?
a. Sensorimotor stage
c.
Formal- operational stage
b. Pre-operational stage
d.
Concrete- operational stage
27. Based on Piagets theory, what should a teacher provide for children in the
concrete operational stage?
a. Learning activities that involve problems of classification and
ordering.
b. Games and other physical activities to develop motor skills.
c. Stimulating environment with ample objects to play with.
d. Activities for hypothesis formulation.

28. Bruners theory on intellectual development moves from enactive to iconic


and symbolic stages. In which stage(s) are diagrams helpful to accompany
verbal information?
a. Enactive and iconic
c.
Symbolic
b. Symbolic and enactive
d.
Iconic
29. Which theory stresses the importance of using pre-reading strategy that
encourages students to use their own experiences to predict and evaluate
the problems an action of characters in the stories they read?
a. Metacognition theory
c. Vocabulary theory
b. Text structure theory
d. scheme theory
30. Vygotsky claimed that social interaction is important for learning. What does
this imply?
a. Children are independent problem solvers.
b. Children learn from adults and other children.
c. Children learn well by passive presentation of information
d. Since they are not capable of instruction, children in the crib have no
learning yet.

31. Which of the following is the most important contribution of Gestalt


Psychology to the theories of learning?
a. cognitive insight
b. Concept readiness in learning
c. Use of multimedia approaches
d. Importance of reinforcement in the learning process
32. Which principle is observed by Ausubels schema theory?
a. Learners have stock knowledge of things based on background
information and experience.
b. There is no need to provide background information.
c. Teachers must presume that learners know everything
d. Children can be taught how to study.
33. What functions are associated with the right brain?
a. Visual, intuitive, logical
b. Visual, non-verbal, logical
c. Visual, intuitive, non-verbal
d. Visual, logical, detail oriented
34. Which learning principle marks the highlight of Multiple Intelligences?
a. Intelligence is not measured in one form
b. Cognitive theory is stressed in learning
c. Learning is static and permanent
d. People have different IQ level
35. If I believe that the childs mind is TABULA RASA, in which process/es will I
engage the child for him/her to learn?
a. Sensory impressions
b. Metacognition
c. Reflections
d. reasoning
36. Paolo, a grade 1 pupil, is happy when he wins in a game but sulks when he
doesnt. Which does Paolos behavior indicate?
a. Egotism
c.
Autonomy
b. Egocentrism
d.
Rigidity of thought
37. James is a shy student and prefers to be alone. Based on Jungs
psychological theory, under what classification does he fall?
a. Extrovert
c.
Introvert
b. Ambivert
d.
Paranoid
38. Which one is Piagets, Freuds and Eriksons thought about childrens play?
a. Makes a childs life so enjoyable that he will tend to hate school life
later.
b. Contributes to the childs mastery of his physical and social
environment.
c. Prepares a child for an excellent academic performance in formal
schooling.
d. Develops in the child a highly competitive attitude because of the
nature of play.
39. Teacher R helped his students recall that stalagmites grow on the ground

while stalactites grow on the ceiling of a cave by associating G in


stalagmites with ground C in stalactites with ceiling. What did teacher R
makes use of?
a. Visual aid
d. Mnemonic device
b. Audio-visual aid
c. Meaning-marker device

40. The use of graphic organizers may tap the brain process because they
mainly _____.
a. Organize the words in some ways the students remember.
b. Appeal to both the left and right hemispheres of the brain.
c. Arouse the students interest in the lesson being learned.
d. Simulate neural wirings of the brain in the process.
41. A person, who has had painful experiences at the dentists office may
become fearful at the mere sight of the dentists office building. Which
theory can explain this?
a. Attribution theory
c. Operant conditioning
b. Generalization
d. Classical conditioning
42. Studies in the areas of neurosciences disclosed that human brain has
limitless capacity. What does this imply?
a. Every child is a potential genius
b. Some pupils are admittedly not capable of learning.
c. Pupils can possibly reach a point where they have learned
everything.
d. Every pupil has his own native ability and his learning is limited to this
native ability.
43.

What does the social constructivist approach emphasize? The students


need __________.
a. opportunities to learn with the teacher and more skilled peers.
b. to be placed in a conductive learning environment
c. to be rewarded with every good performance.
d. to be filled because of empty minds.

44. It is easy for children to learn language because each person has a
Language Acquisition Devices that predispose one to acquire language.
This theory is espoused by __________.
a. Piaget
c.
Gardner
b. Watson
d.
Chomsky
45. To arouse in them the spirit of nationalism, teacher F plays Philippine folk
music as pupils enter the classrooms after flag ceremony. To what theory
does Teacher F adhere?
a. Humanistic psychology
c.
Behaviorism
b. Gestalt psychology
d.
Psychoanalysis
46. Which theory backs up the use of computer assisted instruction?
a. Classical conditioning
b. Insight learning
c. Associative learning
d. Operant learning
47. In a treatment for alcoholism, Ramil was made to drink an alcoholic
beverage and then made to ingest drug that produces nausea. Eventually,
he was nauseated at the sight and smell of alcohol and stopped drinking
alcohol. Which theory explains it?
a. Associative learning
c.
Operant conditioning
b. Social learning Theory
d.
Attribution theory
48. Professor B asserts that a learners behavior must be analyzed not as

separate incidents of stimulus-response but as an organized pattern. To


which theory does Professor B adhere?
a. Gestalt psychology
c.
Instrumental psychology
b. Humanistic psychology
d.
Classical conditioning
psychology

49. Which is a classroom application of the theory on operant conditioning?


a. Reinforce a good behavior to increase the likelihood that the learner
will repeat the response.
b. Help students see the connectedness of facts, concepts and
principles.
c. Create a classroom atmosphere that elicits relaxation.
d. Make students learn by operating manipulatives.
50. To make up for the poor quality of the content of his research report, John
passes a report that is ornately presented in an expensive folder. Which
Filipino trait is illustrated?
a. Psychoanalysis
c.
Humanistic psychology
b. Gestalt psychology
d.
Behaviorism
51. J.B Watson relates that a particular behavior is determined by
environmental factors. Which of the following situations can help the
students develop a positive attitude towards mathematics?
a. Tell students that Math is a difficult subject.
b. Give difficult problems to challenge the students
c. Present lessons from the easiest to the most complex.
d. Have favorable learning atmosphere in the classroom.
52. The psychoanalytic theorist believes the following EXCEPT
a. Development is not in any way colored by emotion.
b. Early experiences with parents extensively shape our development.
c. We have to analyze the inner workings of the mind to understand
behavior
We analyze the symbolic meanings of behavior and not to focus on
d. the behavior.
53. Ruben is very attached to his mother and Ruth to her father. In what
development stage are they according to Freudian psychological theory?
a. Pre-genital stage
c.
Annal stage
b. Oedipal stage
d.
Latent stage
54. Based on Freuds psychoanalytic theory, which component(s) of personality
is(are) concerned with a sense of right and wrong?
a. Super-ego and Ego
c.
Ego
b. Super-ego
d.
Id
55. Teacher D is convinced that whenever a student performs a desired
behavior, provide him/her reinforcement and soon the student learns to
perform the behavior in his/her own. On which principle is the conviction
based?
a. Environmentalism
c. Constructivism
b. Behaviorism
d. Cognitivism
56. Which educational issue can be clarified by understanding Maslows Needs
theory?
a. The effect of poverty on academic achievement
b. The effects of different classroom structures
c. Delinquency in the public schools
d. Sex education issues in school
57. Teacher R, a science teacher, makes sure all eyes are on her as she

demonstrates the proper behavior for lighting Bunsen burner. Whose theory
supports Teacher R practice?
a. Piagets
c. Glassers
b. Vygotskys
d. Banduras
58. Ramil helps his classmates carry heavy books if their teacher sees him and
praises Ramil. Based on Kohlbergs theory of development, in which level of
moral development is Ramil?
a. Conventional
c.
Non- conventional
b. Pre-conventional
d.
Post- conventional
59. A child who gets punished for stealing candy may not steal again
immediately. But this does not mean that the child may not steal again.
Based on Thorndikes theory on punishment and learning, this shows that
_________.
a. Punishment weakens a response.
b. Punishment removes a response.
c. Punishment strengthens a response.
d. Punishment does not remove a response.
60. Behavior followed by pleasant consequences will be strengthened and will
more likely to occur in the future. Behavior followed by unpleasant
consequences will be weakened and will be most likely to be repeated in
the future. Which one is explained?
a. Thorndikes law of effect
b. Banduras social learning theory
c. Freud psychoanalytic theory
d. B.F. Skinners Operant conditioning theory
61. If you want a child to eliminate an undesirable behavior, punish him. This is
accordance with Thorndikes law of __________.
a. punishment
c.
reward
b. exercise
d.
effect
62. A student has just learned a particular formula to solve Math problem. She
tries it with another problem and succeeded in it. So, this student is even
more encouraged to discover easier ways to solve math problems. Which of
Thorndikes laws is depicted in this situation?
a. Readiness
c. Practice
b. Repetition
d. Effect
63.. Miss Beth Burbon, a preschool teacher sees to it that children are given
opportunity to explore and work on different materials so they will develop
initiative and not guilt. She is guided by the theory of ____________.
a. Maslow
c.
Gardner
b. Erikson
d.
Kohlberg
64. Erikson claims that the general attitudes an individual learns during
babyhood persist throughout life and color his perceptions of people and
situation. Which fact about human development does this point to?
a. All individuals are different.
b. Early foundations are critical.
c. Development is affected by culture.
d. Development follows a definite predictable pattern.

65. A teacher should consider the forces that affect the behavior of the student
both from the external environment and what the student is thinking and
feeling from within. This helps her understand the learner and provide more
effective ways to help the child learn. On which theory is this thinking
based?
a. Gestalt theory
b. Lewins field theory
c. Pavlovs conditioning theory
d. Skinners operant conditioning theory

66. Daniel Golemans theory on emotional intelligence highlights the role of


emotion in the success or happiness of an individual. In which situation can
the teacher best cultivate empathy in case students are fighting in the
classroom?
a. Tell the students to stop fighting so that there will be peace in the
classroom.
b. Make them realize how fighting negatively affects themselves and
others.
c. Reprimand the students so that others will not follow the misbehavior.
d. Establish roles and responsibilities to avoid arguments among them.
67. Bernadette, a senior high school student, claims that what she does is what
her conscience dictates as right. Based on Kohlbergs theory on moral
development, in which moral development stage is Bernadette?
a. Punishment and obedience stage
b. Law and order stage
c. Universal ethics stage
d. Social contract stage
68. Who claimed that children are natural learners and therefore, must be
taught in natural settings?
a. Froebel
c. Kohlberg
b. Piaget
d. Montessori
69. Teacher F is convinced that whenever a student performs a desired
behavior and provided reinforcement, soon the student will learn to perform
the behavior on his own. On which principle is Teacher Fs conviction
based?
a. Cognitivism
c.
Constructivism
b. Behaviorism
d.
Environmentalism
70. In social studies class, Teacher I presents a morally ambiguous situation
and asks his students what they would do: On this theory is what Teacher Is
technique based?
a. Kohlberg
c.
Piaget
b. Bandura
d.
Bruner
71. Which theory is demonstrated by this?
A young boy might observe his fathers aggressive outburst and hostile
interchanges with people; when observed with his peers, the young boys
style of interaction is highly aggressive.
a. Classical conditioning
b. Operant conditioning
c. Social cognitive theory
d. Cognitive development theory
72. Which teaching is founded on Banduras social learning theory?
a. Lecturing
c.
Questioning
b. Modeling
d.
Inductive Reasoning
73. Which is a characteristic of a dyslexic child?
a. out of focus
b. Hyperactive

c. Attention-getter
d. Hard up in writing and spelling

74. A student dislikes math due to a traumatic experience with a Math teacher in
the past. Which law explains this?
a. Analogy
c.
Partiality activity
b. Vividness
d.
Disposition mindset
75. Miss Rillo is teaching three-year old boy to put on his shirt. She might first
reward him for placing his right arm in right sleeve, then the left arm in the
left sleeve, the buttoning the front of the shirt, and then tucking the shirt into
his pants. This technique is called
a. fading
c.
conditioning
b. chaining
d.
reinforcement

PAUSE FOR CHECKING AND ITEM ANALYSIS


PART III. (Time Frame: 60 minutes)

Topic: Principles of Teaching, Setting Instructional Objectives,


Instructional Planning, Management of Change, Methods and
Techniques in Teaching
Directions: Read carefully each item. Then encircle the letter of your answer
for what ever is needed in each item.
1.

What I hear, I forget. What I see, I remember. What I do, I understand.


What does this mean? Pupils learn best when they _______.
a. Work with groups
b Learn independently
c. Pay full attention in class
d Take active part in the learning exercises

2. The principle of individual differences requires teachers to _______.


a. give greater attention to gifted learners
b. provide for a variety of learning activities
c. prepare modules for slow learners in class
d. treat all learners alike while in the classroom
3. What does DEVELOPMENTALLY APPROPRIATE PRACTICE type of
schooling focus on?
I. Typical developmental patterns of children
II. Uniqueness of each child
III. Direct Instruction
a. I & II
b. II& III

c. I & III
d. I only

4. Kindergarten and elementary grade pupils are often engaged in various


games as their learning activities. What principle is best observed in the
situation?
a. Children need games to relax and enjoy.
b. Teachers need to relax while the children are at play.
c. Games as a formulized expression of play are natural.
d. Games provide a wide range of social and cognitive experience.
5. Individuals differ from each other in their needs, interests, and abilities, at any

given point in their learning style. Which type of instructional program is


needed by the learners at risk?
a. Adaptive instruction
b. Remedial instruction
c. Traditional instruction
d. Computer-assisted instruction
6.

Mr. David usually holds the Physical Education classes in the open space of
the school in the absence of covered court or gymnasium. One afternoon,
after the warm up activity and brief introduction of the new activity, it started
to rain. Which is the best way to handle the situation?
a. Continue the activity since the students are in their P.E. uniforms.
b. Move the class to their room and give a quiet game
c. Dismiss the class and continue next meeting.
d. Stop the lesson and wait until the rain stops.

7. Every time a new unit is taken up, the teacher presents students with an
advance organizer. Which principle does the teacher apply?
a. Arrange for appropriate practice
b. Assist students to learn communication skills
c. Provide for correct responding on the first trial
d. Organize material into appropriate learning units
8. Which of the following situations best shows the teachers consideration of
the learners developmental patterns?
a. Teaching from the basic to the complex
b. Using the best strategy for the topics
c. Having outdoor activities for learning
d. Developing learners hidden abilities
9.

Which one will most likely increase student participation?


a. Feeling or emotions are not permitted in the discussion
b. The group leader allows quiet members to remain quiet
c. Repeat directions over and over until everyone listens
d. The teacher models good listening habit

10

What does the principle of individual differences require teachers to do?


a. Give less attention to gifted learners.
b Treat all learners alike while teaching.
c. Provide for a variety of learning activities.
d. Prepare modules for slow learners in class.

11. Learning takes place best when


a. learning exercise involve both sides of the brain
b. learning exercise are focused on cognitive objectives
c. learning exercise is focused on the right side of the brain
d. learning exercise are focused on the left side of the brain
12.

Which practice negates teachers role as facilitator of learning?


a. Teacher does more talk so learners talk less.
b. Teacher does less talk for learners to talk more.
c. Teacher makes use of interactive teaching strategies.
d. Teacher caters to multiple intelligences in the classroom.

13. Mrs. Ramos provided reading materials of varying levels of difficulty to the

three groups of learners. Which principle is implemented by her action?


a. Focus on individuals need to achieve
b. Help each student attain goals
c. Provide informative feedback
d. Focus on student attention
14. A reading teacher sees positive effects in the use of various sections of a
newspaper. In what way does the use of the newspaper affect the students?
a. Provides for meeting pupis different interests
b. Contributes to pupils understanding
c. Reinforces learning experiences
d. Strengthens learning skills
15. The primary objective of my lesson is: To add similar fractions correctly.
Before I can do this, I must first aim at this specific objective: To distinguish
a numerator from a numerator. What kind of objective is the latter?
a. Major
c. Primary
b. Terminal
d. Enabling
16. Which behavioral term describes a lesson outcome in the highest level of
Blooms cognitive domain?
a. Create
c. Analyze
b. Evaluate
d. Design
17.

Using the six descriptions of the elements of a good short story. IDENTIFY
IN WRITING THE SIX ELEMETNS IN THE STORY BY O. HENRY with
complete accuracy. The capitalized words are referred to as the
a. Minimum acceptable performance
b. Performance statement
c. Criterion of success
d. condition

18.

Which item is in the highest level of Blooms taxonomy of objectives?


a. Explain how a tree functions in relation to the ecosystem
b. Rate three different methods of controlling tree growth
c. Explain how trees receive nutrients
d. List the parts of the tree

19. Which question is in the highest level of Blooms taxonomy of objectives in


the cognitive domain?
a. What could have happened if there was no control group?
b. What two groups are present in an experimental design?
c. Distinguish between control and experimental group.
d. Judge the validity of the experimental design.
20. In Krathwohls taxonomy of objectives in the affective, which is most
authentic?
a. Characterization
c. Responding
b. Organization
d. Valuing
21. The following are used in writing performance objectives EXCEPT
a. integrate
c. delineate
b. diagram
d. comprehend
22. Below ere questions that must be considered in developing appropriate
learning activities/ experiences EXCEPT one. Which is it?
a. Can the experiences benefit the pupils?

b. Do the experiences encourage the pupils to inquire further?


c. Do the experiences save the pupils from learning difficulties?
d. Are the experiences in accordance with the life patterns of pupils?
23. Based on Magers approach in writing lesson objectives, which objective is
NOT written correctly?
a. At the end of the period, 80 % of the class is able to compose a
seven to ten sentences paragraph that observes unity and clarity.
b. At the end of the period, the student is able to compose a seven-toten sentence paragraph that observes clarity and unity.
c. At the end of the period, 80 % of the class is able to solve 5 word
problems with 90 % accuracy.
d. At the end of the period, 80% of the class is able to solve the 5 word
problems correctly.
24. Which among the following objectives in the psychomotor domain is the
highest in level?
a. To contract a muscle.
b. To run a 100-meter dash.
c. To dance the basic steps of the waltz.
d. To distinguish distant and close sounds
25. Which statement on instructional panning is wrong?
a. The experienced teacher needs a lesson plan.
b. A written lesson plan gives focus to a teachers lesson plan.
c. A well-written lesson plan is a guarantee to effective teaching.
d. The lesson plan serves as a road map for the teachers lesson.
26. As a teacher, what do you do when you engage yourself in major task
analysis?
a. Test if learning reached higher level thinking skills.
b. Breakdown a complex task into sub-skills.
c. Determine the level of thinking involved.
d. Revise lesson objectives.
27 The steps of the goal-oriented instructional model arranged in order include
a. pre-assessment, specification of objectives, instruction and evaluation
b. pre-assessment, specification of objectives, motivation and instruction
c. specification of objectives, pre-assessment, evaluation
d.. specification of objectives, instruction and evaluation
28. The reading teacher stays 30 minutes daily after class and teaches the slow
readers how to read. What is the practice called?
a. Make up class
c. Remedial instruction
b. Enrichment activity
d. Developmental reading
29. Which is the best reason why Mr. Cruz begins a lesson in Mathematics by
checking and reviewing the previous days assignment and provides
practice and drills?
a. make sure that students understand the pre-requisite skills for the
lesson
b. Give the students the opportunity to answer workbook exercises.
c. Make learning interesting and enjoyable for students
d. Prepare the students for the mastery tests
30. Mr. David usually holds the Physical Education classes in the open space of
the school in the absence of covered court or gymnasium. One afternoon,

after the warm up activity and brief introduction of the new activity, it started
to rain. Which is the best way to handle the situation?
a. Continue the activity since the students are in their P.E. uniforms.
b. Move the class to their room and give a quiet game
c. Dismiss the class and continue next meeting.
d. Stop the lesson and wait until the rain stops.
31. Planning for instruction should be based on certain developmental principles
because the
a. Lesson objectives will determine the students activities in class.
b. Students need to learn the lessons using appropriate materials.
c. Class activities must be appropriate to the students background.
d. Teaching job must be procedural/systematic in order to be effective.
32. Which is NOT a basic component of a lesson plan?
a. Assignment
c. Resources
b. Evaluation
d. Content
33. Which statement about lesson plan is LEAST accepted?
a. Lesson plan should be in constant state of revision.
b. A good daily lesson plan ensures a better discussion.
c. Students should never see a teacher using a lesson plan.
d. All teachers regardless of their experience should have daily lesson
plan.

34. A catalyst teacher is good at the following skills, EXCEPT:


a. makes effective use of cooperative learning techniques
b. Ask through-provoking questions.
c. Require uniform reports
d. Do inquiry techniques
35. A school Division Superintendent was enthusiastically lecturing on the
benefits and demands of Accreditation Program for Public Elementary
Schools. (APPES). Thinking of its many demands, most of the school heads
were not very happy about it and the older ones were whispering we have
every reason to retire soon.
What does this tell about the change process?
a. Leadership can effect the desired change in persons despite
opposition from the persons themselves.
b. Resistance to change is insurmountable
c. People resist change for no reasons
d. People tend to resist change
36. For an effective presentation of Article XIV of the Philippine Constitution,
which is MOST appropriate?
a. Team Games
c. Independent study
b. Learning circles
d. Direct instruction
37.
I. With indirect instruction in mind, which does NOT belong to the group?
a. Experiential method
c. Discovery method
b. Lecture recitation
d. Inductive method
38.
1. I want the fast learners in my class to do selfdirected learning. What
strategy will I use?
a. Metacognition
c. Collaborative learning
b. Social dialogue
d. Problem based learning
39.
2. My problem is that there are too many topics to cover and I may not able to
finish them all before classes end in March. Which approach when used
may help solve my problem?
3.
a. Thematic
c. Direct instruction
b. Constructive
d. Experimental approach
40.
4. If you make use of the indirect instruction method, you begin your lesson
with _________
5.
a. Guided practice
b. Independent practice
c. A review of previous days work
d. Advance organizers that provide an overall picture of the lesson
41.6. In self-directed learning, to which extent should a teachers scaffolding be?
7.
a. To the degree the student needs it.
b. None, to force the student to find his way and learn faster by himself
c. To the maximum, in order to extend to the student all the help he
needs
d. To the minimum, to speed up the development of the students sense
of independence

42. I want to use the direct instruction strategy. Which sequence of steps will I
follow?
I.
Reviewing the previous days work
II.
Independent Practice
III.
Guided student practice
IV.
Presenting and structuring
V.
Feedback
a.
b.
c.
d.

I IV III II V
I IV II III V
IV I III II V
IV I II III V

43. To make our children little scientists, which method(s) should we employ
more often?
II.
Inquiry
III. Problem solving
IV. Laboratory
a. II and III
c. I and III
b. I and II
d. I, II, and III
44.

For a more authentic application and evaluation of learning, which method


is recommended?
a.
Inquiry
c.
Laboratory
b.
Project
d.
Question and answer

45. To encourage introspection, which teaching method is MOST appropriate?


a. Process
c. Reflective
b. Cognitive
d. Cooperative learning
46. To give opportunity for bonding and sharing, which can be of help?
The use of ____________
a. Discovery approach
c. Constructivist approach
b. Exploratory approach
d. Cooperative learning approach
47.
V. To teach your students how to cook a recipe effectively, which method
should you use?
a. Demonstration
c. Discovery
b. Questioning
d. Unit
48.

The following are characteristics of interdisciplinary teaching EXCEPT:


a. Allows learners to see connectedness between things
b. Provides learning opportunities in a real-world context
c. Eliminates boundaries between content area
d. Discussion from a single perspective

49. I started my lesson with a graphic presentation of data then asked my


students to analyze and interpret them and draw implications from them.
How did my lesson development proceed?
a. Inductively
b. Deductively
c. Transductively
d. Inductively and deductively
50. Nicole enjoys games like scrabble, anagrams, and password. Which type of
intelligence is strong for Nicole?

a.
b.
c.
d.

Logical and mathematical intelligence


Interpersonal intelligence
Linguistic intelligence
Spatial intelligence

51. Which activity should a teacher have more for his students if he wants them
to develop logical-mathematical thinking?
a. Problem solving
c. Storytelling
b. Choral Reading
d. Drama
52. For whom is individual research most effective? The ____ intelligent group.
a. naturalist
c. Interpersonally
b. existentialist
d. Intrapersonally
53.

Knowledge is innate in the soul. Provoke people into thinking for


themselves rather than teach them what they did already know, said one
Greek philosopher. What is the application of this in the classroom today?
Use of ______.
a. Cooperative learning
d. Concept mapping
c. Questioning method
b. Role playing

54. In which of the following is team teaching done?


a. The P.E and Health teacher and the Music teacher divide the school
calendar into two and teach their subject separately.
b. The P.E and Health teacher and the Music teacher prepare separate
course syllabi but agree on what each one should teach.
c. The P.E and Health teacher and the Music teacher work on an
integrated course syllabus and take turn when they teach.
d. The P.E and Health teacher and the Music teacher are in the same
classroom when they teach PHEM and follow an integrated syllabus.
55. If a teacher plans a constructivist lesson, what will he most likely do? Plan
how he can _____
a. engage his students in convergent thinking
b. evaluate his students work
c do reciprocal teaching
d. lecture to his students
56. Teacher Ms pupils are quite weak academically and his lesson is already
far behind time table. How should Teacher M proceed with his lesson?
a. Experientially
c. Inductively
b. Deductively
d. Logically
57. Which method has been proven to be effective in courses that stress
acquisition of knowledge?
a. Socratic method
c. Indirect Instruction
b. Mastery learning
b. Cooperative learning
58. By what name is Indirect instruction or the Socratic method also known?
a. Indirect Method
c. Morrison method
b. Mastery learning
d. Questioning method
59. Which is a form of direct instruction?
a. Problem solving
b. Discovery process

c. Inductive reasoning
d. Programmed instruction

60. Teacher H gave her first-grade class a page with a story in which pictures
take the place of some words. Which method did she use?
a. The language experience approach
b. The whole language approach

c. The Spaulding method


d. The rebus method
61. I combined several subject areas in order to focus on a single concept for
interdisciplinary teaching. Which strategy/method did I use?
a. Problem-entered learning
c. Thematic instruction
b. Reading-writing activity
d. Unit method
62. With direct instruction in mind, which does NOT belong to the group?
a. Inductive reasoning
c. Lecture-recitation
b. Problem solving
d. Discovery
63. In self-directed learning, to what extent should teachers scaffolding be?
a. To a degree the student needs it.
b. None, to force the student to learn by himself.
c. To the maximum, in order to extend to the student all the help he
needs.
d. To the minimum, to speed up development of students sense of
independence.
64. Which may work against collaborative teaching?
a. Joint rewards
b. Homogeneous grouping
c. Establishment of mutual goals
d. Knee-to-knee seating arrangement by group
65. Which method makes possible hands-on, minds-on learning?
a. Unit
c. Integrative
b. Project
d. Demonstration
66. Which holds TRUE to the deductive method?
a. Less subject matter covered
b. Teacher-dominated
c. Student-dominated
d. Highly interactive
67. I use the gumamela to teach the parts of a complete flower. Later, the
flowers will be studied in comparison to the gumamela. What teaching
method did I use?
a. Problem-solving method
c. Type study method
b. Laboratory method
d. Discovery method
68. Which term refers to a teacher helping a colleague grow professionally?
a. Technology transfer
c. Peer mentoring
b. Independent study
d. Facilitating
69. Which activity should a teacher have more for his students if he wants them
to develop mathematical thinking?
a. Choral reading
c. Story telling
b. Problem solving
d. Drama
70. Speaking of learning style, the mastery learner learns best from _____.
a. drill
c. debates
b. dramas
d. role playing

71. With individualized teaching strategies as point of reference, which does not
belong to the group?
a. Journal writing
b. Independent study
c. Demonstration method
d. Computer-assisted instruction
72. When teacher attempts to develop his students metacognitive skills, he
teaches them to _____.
a. visualize
c. formulate hypothesis
b. recall past lesson
d. think about their thinking

73. The reading teacher stays 30 minutes daily after class and teaches the slow
readers how to read. What is the practice called?
a. Make up class
c. Remedial instruction
b. Enrichment activity
d. Developmental reading
74. Mrs. Santos aims to maximize individual learning among her third year
students in Science. If she intends to permit students to proceed at their
own rate, which of the following will she apply?
a. Cooperative learning
c. Hands-on learning
b. Modular instruction
d. Supervised study
75. The human evocative approach and the 4 As end with _____.
a. Generalization
c. Application
b. Abstraction
d. Analysis
76. The process of thinking about ones thinking to develop critical thinking is
known as _____.
a. Metacognition
c. Reflection
b. Hypothesizing
d. Meditation
77. The K-W-L strategy performs the following functions, EXCEPT;
a. It encourages the reader to interact with the text.
b. It provides summary of what has been learned.
c. It provides purpose for reading.
d. It activates previous schema.
78. Science Teacher Z introduces a topic on earthquakes. Then she helps her
students determine what they need to know about earthquakes and the
activities and resources which help them learn. Which technique does
Teacher Z employ?
a. Computer-assisted instruction
b. Individualized instruction
c. Self-directed learning
d. Collaborative learning
79. Which is a characteristic of infused or integrated instruction?
a. Includes all thinking process from the low level to high level
b. Does not allow deviation from intended objective
c. Confines itself to higher level thinking process
d. Exclude related topics
80.

Which is correct statement about teaching method?


a. There is a best method in teaching
b. There is one best method for teaching
c. Indirect instructional method is teacher-centered
d. No one teaching method can accomplish every goal

81. The lesson is on the PROS and CONS of having a federal government. Mr.
Reyes wants to do high level thinking and to develop a view of the federal
government from different perspectives. Which technique will be most
appropriate?
a. Panel discussion
c. Simulation
b. Role playing
d. Debate
82. I want my students to overlearn what they have learned at the same time

help others to master the lesson. Which activity is BEST?


a. Peer tutoring
b. Simulation

c. Practice
d. Game

83. If I want to hone my students meta-cognitive ability, which is MOST fit?


a. Drill
b. Debate

c. Journaling
d. Brainstorming

84. Which technique in cooperative learning is used when new information is


divided equally among all group members and each student teaches his/her
portion to the other group members?
a. Jigsaw
b. Roundtable

c. Roundrobin
d. Think pair share

85. I want to have small group discussions in my class. Which topics will be
best for group discussions? Those topics _______.
a. where a high degree of consensus among students does not yet exist
b. where a high degree of consensus among students already exist
c. that are formally structured text
d. that are factual
86. To teach democratic process to the pupils, School A decided to pattern the
election of class and school officers after the local election process. There
were classifications set for candidates, limited period of campaign and rules
for posting campaign materials, etc. Which of the following did the school
employ?
a. Philips 66
c. Symposium
b. Simulation
d. Role playing
87. By the use of a technique, I learn that my students are quite weak in
organizing and integrating concepts. Which technique must I have used?
a. Game
c. Brainstorming
b. Simulation
d. Semantic webbing
88. My lesson is on methods of family planning. I wanted my class to gain a
holistic understanding of family planning methods. So I invited a priest to
talk on morality of family planning methods, a doctor on the physiological
aspect of family planning, an official of Population Commission for his
experiences as one who practices family planning to talk before my class
and to answer questions raised by the class. What techniques did I employ?
a. Forum
c. Symposium
b. Debate
d. A Panel discussion
89. Mr. Alcoriza wants to teach his pupil the technique on reading for
information. Which technique should he use?
a. Prior knowledge
c. Story map
b. Text structure
d. SQ3R
90. Teacher Lawrence wants group members to give their full attention to what
one individual wants to express. This is done by the whole group seated in a
circle while 2 chairs are placed in the center of the circle for two members to
converse while the others listen. What is this technique called?
a. Round table
c. Symposium
b. Forum
d. Fish bowl technique

91. If teacher wants to develop in the learner the skills to organize ideas, which
one should be employ?
a. Histogram
c. Graphic organizer
b. Venn diagram
d. K-W-L technique
92. Which visual display is a result of students comprehension of a selection
read proven by his ability to organize and integrate concepts and
information gleaned from the selection?
a. Flowchart
c. Venn Diagram
b. Story map
d. Semantic Web

93. Which graphic organizer is used to show how a series of events interact to
produce a set of results again and again?
a. Series of events chart
c. Cycle
b. Fishbone Diagram
d. Timeline
94. The teachers first task in the selection of media in teaching is to determine
the
a. choice of the students
b. availability of the media
c. objectives of the lesson
d. technique to be used
95. Based on Edgar Dales Cone of Experience, which activity is closest to the
real thing?
a. View images
c. Watch demo
b. Attend exhibit
d. Hear
96. Which criterion should guide a teacher in the choice of instructional
devices?
a. Cost
c. Attractiveness
b. Novelty
d. Appropriateness
97. Under what category will a globe as an instructional material fall?
a. Mock up
c. Model
b. Picture
d. Real
98. The brain of the computer is the
a. CPU
b. mouse
99.

c. monitor
d. keyboard

For effective classroom management, when should a teacher undertake


the task of setting up of routine activities?
a. Every homeroom day
b. On the very first day of the school
c. Everyday at the start of the session
d. As soon as the students have adjusted to their schedules

100.. For counseling to be successful, which assumption must be AVOIDED?


a. The environment must provide assurance of confidentiality
b. The counselor must be able to relate to the student
c. The student is willing to participate in the process
d. The councilor tell the students what to do
PAUSE FOR CHECKING AND ITEM ANALYSIS

PART IV. (Time Frame: 60 minutes)

Topic: Measurement and Evaluation


1. Mr. Reyes wants to test students ability to work with and relate to people.
Which will be the most authentic assessment procedure to give to his
students?
a. Writing an essay on how to develop human relation skills
b. Writing articles on working and relating to people
c. Organizing a community project
d. Conducting home visitation
2. While teaching how to compute the area of quadrilaterals, Teacher Joana
checks whether her students understand what she is teaching. What is
Teacher Joana engaged in?
a. Criterion referenced
c. summative evaluation
evaluation
b. Norm-referenced evaluation
d. formative evaluation
3. Mrs. Zamora discovered that her pupils are weak in comprehension. To
further determine in which particular skill(s) her pupils are weak, which test
should she give?
a. Placement Test
c. Standardized Test
b. Diagnostic Test
d. Aptitude Test
4. Quiz is to formative test while periodic is to _____.
a. criterion-referenced test
c. summative test
b. norm-reference test
d. diagnostic test
5.

What type of test is given to determine the admission and non-admission of


a student to the program?
a. Aptitude
c. Diagnostic
b. Placement
d. Achievement
In the context of the theory of multiple intelligence, one weakness of the
paper-pencil test is
a. It puts non-linguistically intelligent at a disadvantage.
b. It is not easy to administer.
c. It utilizes so much time.
d. It lacks reliability.

7. Under which assumption is portfolio assessment based?


a. Portfolio assessment is a dynamic assessment.
b. Assessment should stress the reproduction of knowledge.
c. An individual learner is adequately characterized by a test score.
d. An individual learner is inadequately characterized by a test score.
8.

Which statement on IQ test is WRONG?


a. Measures fixed potential
b. Measures current performance
c. Can be advanced by maturational changes
d. Can be enriched by environmental experiences

9. Which are direct measures of competence?

a. Personality test
b. Performance tests

c. Standardized tests
d. Paper-and-pencil tests

10. Which is an advantage of teacher-made test over standardized tests?


a. better adapted to the needs of the pupils
b. more objectively scored
c. highly reliable
d. highly valid
11. Which is a good practice that a teacher uses in correcting students errors?
a. Encourage the students to read books and magazines to improve
their spelling
b. Readily correcting the error upon utterance to distract the students
flow of thought.
c. Ignore the mistakes; anyway she will commit the same error in the
future.
d. Noting students error and dealing with them after the class activities.
12. Which term refers to the collection of students products and
accomplishments for a period for evaluation purposes?
a. Diary
b. Portfolio
c. Anecdotal record
d. Observation report
13. Which statement about performance-based assessment is FALSE?
a. It emphasizes merely process.
b. It stresses on doing, not only knowing.
c. It accentuates on process as well as product.
d. Essay tests are an example of performance-based assessments.
14. Which can effectively measure students awareness of values?
a. Projective techniques
c. Moral dilemma
b. Anecdotal record
d. Likert scales
15. These are significant information about a student, gathered through the use
of various techniques, assembled, summarized and organized in such a
way that they may be used effectively. What is referred to?
a. Cumulative record
c. Test profiles
b. Personal inventory
d. Case studies
16. Which is a norm-referenced statement?
a. Danny spelled 35 words out of 50 correctly.
b. Danny was able to spell 90% of the words correctly.
c. Danny performed better in spelling than 60% of his classmate.
d. Danny was able to spell 90% of the words correctly and spelled 35
out of 50 correctly.
17. Teacher Z is engaged in a criterion-referenced interpretation of scores.
Which of the following does she do?
a. She describes every individual student performance in relation to the
performance of the age group of student.
b. She describes every individual student performance in relation to the
clearly defined learning task.
c. She uses a specified population of persons as its interpretative frame
of reference.
d. She compares every individual student performance with the
performance of the test.

18. NSAT and NEAT results are interpreted against set mastery level. This
means that NSAT and NEAT fall under
a. criterion-referenced test
c. intelligence test
b. norm-referenced test
d. aptitude test

19. Which test has broad sampling of topics as strength?


a. Short answer test
c. Performance Test
b. Multiple Choice
d. Essay test
20. Which multiple choice type of test has options that are based on a graphical
presentation of data or on paragraph or picture?
a. Structured response test
c. Contained-options test
b. Setting and option test
d. Stem-and-options test
21.. Teacher gives this item to his class for assessment: CORE IS TO EARTH
AS NUCLEUS IS TO ATOM. THEN SHE ASKS THE STUDENTS TO
EXPLAIN HOW THE RELATIONSHIP BETWEEN CORE AND EARTH IS
SIMILAR TO THE RELATIONSHIP BETWEEN ODOMETER AND
DISTANCE. What classroom assessment technique is this called?
a. Webbing
c. Metaphor
b. Analogy
d. Simile
22. The strongest disadvantage of the alternate-response type of test is
a. the encouragement of rote memory
b. the demand for critical thinking
c. the high possibility of guessing
d. the absence of analysis
23. Which does NOT belong to the group?
a. Matching
c. Multiple choice
b. Completion
d. Alternate response
24. With synthesizing skills in mind, which has the highest diagnostic value?
a. Essay test
c. Performance test
b. Completion test
d. Multiple choice test
25. To evaluate teaching skills, which is the most authentic tool?
a. Observation
c. Short answer test
b. Non-restricted test
d. Essay test
26. What criterion (a) is (are) applied by a principal who evaluates teachers
based on the result of achievement tests of pupils?
a. Product attitude
c. Product
b. Process
d. Attitude
27. Which is the best source of information on teachers performance?
a. Self-evaluation
c. Students evaluation
b. Peer evaluation
d. Supervisory evaluation
28. When is a test item valid?
a. When it measures the trait that it intends to measure
b. When it covers a broad scope of subject matter
c. When the test is easy to administer
d. When scores are reliable
29. Teacher F wanted to teach the pupils the skill to do cross stitching. Her
check up quiz was a written test on the steps of cross stitching. Which
characteristic of a good test does it lack?
a. Scorability
c. Objectivity
b. Reliability
d. Validity
30 If your Licensure Examination (LET) items sample adequately the
competencies listed in the syllabi, what type of validity does LET possess?

a. concurrent
b. predictive

c. construct
d. content

31. What does concurrent validity require?


a. item analysis
c. correlation study
b. item difficulty
d. peer consultation
32. If all the passers of 2006 Licensure Examinations for Teachers will turn out
to be the most effective in the Philippine school system, what type of validity
does LET possess?
a. content
c. predictive
b. construct
d. concurrent
33. When a significantly greater number from the lower group gets a test item
correctly, what does this imply?
a. The test items is not highly reliable
b. The test item is highly reliable.
c. The test item is not very valid.
d. The test item is very valid.
34.

When is a test considered reliable?


a. If it is easy to score.
b. If it is easy to administer.
c. If it is consistent and stable.
d. If it served the purpose for which it is constructed.

35. Danny takes an IQ test thrice and each time, he earns a similar score. What
does the test possess?
a. validity
c. objectivity
b. reliability
d. scorability
36. Below is a list of method used to establish the reliability of the instrument.
Which method is questioned for its reliability due to practice of familiarity?
a. Split half
c. Equivalent form
b. Test-retest
d. Kuder Richardson
37. Which error do teachers commit when they tend to overrate the
achievement of students identified by aptitude test as gifted because they
expect achievement and giftedness to go together?
a. Central tendency error
c. Severity error
b. Generosity error
d. Logical error

38. Writing an original essay is an example of which level of objective in the


cognitive domain?
a. Analysis
c. Evaluation
b. Synthesis
d. Application
39. A test has an instruction Group the following items according to their
habitat. What thinking skill is this testing?
a. Inferring
c. Standardized Test
b. Classifying
d. Aptitude Test
40. Mrs. Lopez asked the students to write a short paragraph on taking
examinations by observing unity, coherence, emphasis and variety. What
skill does this item test?
a. Evaluate
c. Create
b. Understand
d. Recall
41. In test construction, what does TOS mean?
a. Table of Specifications
c. Terms of Specification
b. Table of Specific Items
d. Table of Specifics
42. Which is the first step in planning an achievement test?
a. Define the instructional objective.
b. Select the type of test items to use.
c. Decide on the length of the test.
d. Build a table of specifications.
43. Out of 3 distracters in a multiple choice test item, namely B, C, and D no
pupil chose D as an answer, This implies that D is _____.
a. a vague distracter
c. an effective distracter
b. a plausible distracter
d. an ineffective distracter
44. Which multiple choice type of test has options that are based on a graphical
presentation of data or on paragraph or picture?
a. Structured response test
c. Contained-options test
b. Setting and option test
d. Stem-and-options test
45. Read this test item then answer the question: PHYSICIAN IS TO HUMAN
BEINGS AS VETERENARIAN IS TO _______.
A. FRUITS
B. ANIMALS
C. MINERALS
D. VEGETABLES
QUESTION: The above test item is an example of a/an ______.
a. stimulus
c. exercise
b. analogy
d. puzzle
46. Here is a sample of TRUE-False test item: ALL WOMEN HAVE A LONGER
LIFE SPAN THAN MEN. What is wrong with the test item?
a. A specific determiner gives clue to the answer.
b. The test is quoted verbatim from a textbook.
c. The test item contains trivial detail.
d. The test item is vague.

47. Read and analyze the matching type of test given below.
Directions: Match column A and column B. Write only the letter of your
correct on the blank of the left column.
A
____1. Baguio
____2. Ninoy Aquino
____3. June 12
____4. Jose Rizal
____5. Mt. Apo

B
a. Independence Day
b. Tallest mountain in the
Philippines
c. The national hero
d. National Heroes Day
e. The summer capital of the
Philippines
f. A senator that turned hero

The test violated which of the following rules?


I.
The statement of directions should specify how the
matching be done.
II.
The shorter part of the test should be in column B.
III.
The items should be homogeneous.
IV.
There should be distracters.
a. II and III only
c. I, II, and IV only
b. I, II, and III only
d. I, II, III and IV
48. What improvement should be done in this completion test item: An example
of dicot is____________.?
a. The blank should be short only.
b. The item should give more clues.
c. There should only be one correct answer.
d. The blank should be at the beginning of the sentence.
49 Heres a test item: Jose Rizal was born in Calamba, Laguna on June ___,
1861. What is WRONG with the item?
a. The blank is very short.
b. The blank is near the end.
c. It concerns itself with trivia.
d. It is open to more than one correct answer.
50

Which is included in item analysis?


a. Determining the percentage equivalent to the cut score.
b. Determining the effectiveness of distracters.
c. Determining the cut off score
d. Identifying the highest score

51. What does a facility index of a test item 0.50 mean?


a. easy item
c. very difficult item
b. very easy item
d. item with moderate difficulty
52. A test item has a difficulty index of 0.85 and a discrimination index of 0.54.
What should the teacher do with this test item?
a. Reject the item
c. make it a bonus item
b. Retain the item
d. exclude the item
53. What does a negative discrimination index mean?
a. There are more from the upper group that got the item right.
b. There are more from the lower group who got the item right.

c. The test item is quite difficult.


d. The test item is quite easy.
54. STUDY THE TABLE ON ITEM ANALYSIS FOR NON-ATTRACTIVENESS
AND NON-PLAUSIBILITY OF DISTRACTORS BASED ON THE RESULT
OF A TRY OUT TEST IN MATH. THE LETTER MARKED WITH AN
ASTERISK IS THE CORRECT ANSWER.
Item No. 1
A*
B
C
D
Omit
Upper 27%
10
4
1
1
0
Lower 27%
6
6
2
2
0
Based on the table, which is most effective distracter?
a. Option D
c. Option A
b. Option C
d. Option B
55. Which of the following is NOT a factor in errors in measurement?
a. The test questions may get outdated.
b. The students may take the test for granted.
c. The practice effect may increase a students score.
d. Administering the test twice may measure different attributes.
56.

Test norms are established in order to have a basis for


a. Planning effective instructional devices
b. Establishing learning objectives
c. Identifying pupils difficulties
d. Comparing test scores

57. The criterion of success in Teacher Lyns objective is that the pupils must
be able to spell 90% of the words correctly. Ana and 19 others correctly
spelled 40 words only out of 50. This means that Teacher Lyn
a. Attained her lesson objective
b. Attained her objective because of her effective spelling drill
c. Did not attain her lesson objective because of the pupils lack of
attention
d. Failed to attain her lesson objective as far as the twenty pupils are
concerned
58. What can be said of Mike who obtained a score of 75% in a Grammar Test?
a. He performed better than 75% of his classmates
b. He answered 75% of the test items correctly
c. He answered 75 items in the test correctly
d. He got a raw score of 75
59.

Which is a type of graph in which lines represent each score or set of


scores?
a. Frequency polygon
b. Scattergram
c. Histogram
d. Scatterplot

60.

Which statement correctly applies to student who got a score of 72 in the


test?
a. He obtained a raw score of 72
b. He answered only 72 items in the test.
c. He surpassed the scores of 72 students
d. He correctly answered 72% of the items in the test

61.

Which statement on test result interpretation is CORRECT?


a. A raw score by itself is meaningful.
b. A students score is a final indication of his ability
c. Test scores do not in any way reflect teachers effectiveness.
d. The use of statistical technique gives meaning to pupils scores.

62. What is the mode of the following score distribution:


96,97,98,97,93,90,89,97,81,80?
a. 98
c. 96
b. 97
d. 83
63. What is the mean of this scores distribution: 4,5,6,7,8,9,10?
a. 7
c. 8.5
b. 6
d. 7.5
64. What measure/s of central tendency does the number 16 represent in the
following data:14,15,17,16,19,20,16,14,16?
a. Mode and median
c. Mode
b. Median
d. Mean
65.

You give 100-point test; three students make scores of 95, 92, and 91
respectively while the other 22 students in the class make scores ranging
from 33 to 67. The measure of central tendency which is apt to describe for
this group of 25 students is
a. the mean
b. the mode
c. the median
d. an average of the median and mode

66. Which measure(s) of central tendency can be determined by mere


inspection?
a. Mean
c. Median
b. Mode
d. Mode & median
67. Teacher C adds the number of cases and 1 over 2 to obtain
a. mode
c. median
b. mean
d. median and mode
68. Which of the following is (are) the least stable measure(s) of central
tendency?
a. Mode
b. Mean
c. Median
d. Mode and Median

69.

Which measure(s) of central tendency separate(s) the top half of the group
from the bottom half?
a. Median and mean
c. Mean
b. Median
d. Mode

70

Which measure(s) of central tendency is (are) most appropriate when the


score distribution is negatively skewed?
a. Mode
c. median
b. Mean
d. mean and mode

71.

Which of the following measures is most affected by an extreme score?


a. Semi-interquartile range
c. Mode
b. Median
d. Mean

72. Which applies when the mean, median and mode are equal?
a. The distribution of the scores is normal.
b. The distribution of the scores is skewed.
c. The distribution of the scores is identical.
d. The distribution of the scores is asymmetrical.
73.

Which of the following could produce more than one value?


a. Mean
c. Median
b. Mode
d. Mean of grouped data

74. Which applies when there are extreme scores?


a. The median will not be very reliable measure of central tendency.
b. The mode will be the most reliable measure of central tendency.
c. The mean will not be a very reliable measure of central tendency.
d. There is no reliable measure for central tendency.
75. The sum of all scores in a distribution is always equals
a. The mean divided by the interval size.
b. The mean times the interval size.
c. The mean divided by the N.
d. The mean times the N.
76. Median is a point as standard deviation is to ____.
a. area
c. volume
b. square
d. distance
77.

STUDY THE TABLE BELOW THEN ANSWER THE QUESTION:


Scores
Percent of Students
0-59
2%
60-69
8%
70-79
39%
80-89
38%
90-100
13%
In which scores interval is the median?
a. In between the intervals of 60-69 and 70-79
b. In the interval 80 to 89
c. In the interval 70-79
d. In the interval 60-69

78.

Referring to assessment of learning, which assessment on the normal


curve is FALSE?

a. The normal curve may not necessarily apply to homogeneous class.


b. The normal curve is sacred. Teachers must adhere to it no matter
what.
c. When all pupils achieve learning as expected, curve may deviate
from the normal curve.
d. The normal curve may not be achieved when every pupil acquires
targeted competencies.
79. Which applies when all high frequencies are NOT adjacent?
a. The score distribution will neither be bimodal nor multimodal.
b. The score distribution will be bimodal or multimodal.
c. There will only be one point of score concentration.
d. There is no point of score concentration.
80.

The score distribution follows the normal curve. What does this mean?
a. Most of the scores are on the -2 SD
b. Most of the scores are on the +2 SD
c. The scores coincide with the mean
d. Most of the scores pile up between -1SD and +1SD

81.

Which is implied by a negatively skewed score distribution?


a. Most of the scores are low.
b. Most of the scores are high.
c. Most pupils are underachievers.
d. The scores are evenly distributed from the left to the right

82.

All the examinees obtained scores below the mean. A graphic


representation of the score distribution will be
a. perfect normal curve
c. positively skewed
b. negatively skewed
d. leptokurtic

83

Which applies when skewness is zero?


a. Mean is greater than the median.
b. Median is greater than the mean.
c. Scores are normally distributed.
d. Scores have three modes.

84. A class is composed of bright students. The scores distribution will most
likely to be
a. platykurtic
b. very normal
c. skewed to the left
d. skewed to the right
85.. Study the results of the different tests given to the class where Romalyn
belongs.
Subjects, Class Mean Score, SD, and Romalyns Score
Subject
Mean
SD
Peters Score
Math
56
10
73
Physics
41
5
51
English
80
16
90
In which subject(s) did Romalyn perform the best in relation to the groups
mean performance?
a. Math
c. English

b. Physics

d. Physics and English

86. Aura Vivian is one-half standard deviation above the mean of his group in
arithmetic and one standard deviation above in spelling. What does this
imply?
a. She excels both in spelling and arithmetic.
b. She is better in arithmetic than in spelling.
c. She is better in spelling than in arithmetic.
d. She does not excel in spelling nor in arithmetic.
87. Q1 is to 25th percentile as median is to _____.
a. 25th percentile
c. 50th percentile
th
b. 40 percentile
d. 75th percentile
88. After scoring, teacher G got the difference of the highest and the lowest
scores in each class. What did she compute?
a. Range of the middle 50%
b. Range of the lowest fourth
c. Standard deviation
d. External range
89. The teacher gives achievement test to his 25 students. The test consists of
the 50 items. He wants to classify his students performance based on the
test result. What is the appropriate measure for the position?
a. Z-value
c. Percentage
b. Stanine
d. Percentile rank
90. In a normal distribution curve, a T-score of 70 is
a. two SDs below the mean
c. one SD below the mean
b. two SDs above the mean
d. one SD above the mean
91.

Ruben scored 60 on a percentile-ranked test. This means that _____.


a. 60% of the students who took the test scored higher than Ruben
b. 60% of the students who took the test scored lower than Ruben
c. Ruben got 60% of the question wrong
d. Ruben got 60% of the questions right.

92. The computed r for English and Math scores is 0.75. What does this mean?
a. The higher the scores in Math the higher the scores in English
b. The higher the scores in Math, the lower the scores in English
c. The lower the scores in Math, the lower the scores in English
d. Math and English scores are not in any way related
93.

Which is an element of norm-referenced grading?


a. An absolute standard
b. The performance of the group
c. The students past performance
d. What constitutes a perfect score

94.

Ms Silva is conscious of severity error in grading her students. What


grading practice will she likely prefer?
a. Numerical grading
c. letter grading
b. Descriptive grading
d. scoring rubric

95.

Teacher M, a department head, wants to ensure comparability of grades of


students in her department. She should be

a.
b.
c.
d.

Using a table of specifications


Giving more high level questions
Determining the level of difficulty of the test
Constructing departmentalized examinations for each subject area

96.

The following are trends in marking and reporting system, EXCEPT


a. indicating strong points as well as those needing improvement
b. conducting parent-teacher conferences as often as needed
c. supplementing subject grades with checklist on traits
d. raising the passing grade from 75 to 80

97. In research, which is another term for independent variable?


a. Response
c. Criterion
b. Outcome
d. Input
98. Which is TRUE to a one-group-posttest-only design?
a. Two groups of subjects that are not statistically equivalent are used in
the experiment.
b. The subjects are randomly assigned to two groups.
c. No pretest is done at the start of the experiment.
d. Pre-test was done prior to the treatment.
99. When points in the scattergram are spread evenly in all directions this
means that
a. the correlation between two variables is positive
b. there is no correlation between two variables
c. the correlation between two variables is high
d. the correlation between two variables is low
100. The analysis of Variance utilizing the F-test is the appropriate significance
test to run between
a. medians
c. two means only
b. frequencies
d. three or more means

Das könnte Ihnen auch gefallen